Exam 1

अब Quizwiz के साथ अपने होमवर्क और परीक्षाओं को एस करें!

You ask a patient to draw a clock. He fills in all the numbers on the right half of the circle. What do you suspect? A) Hemianopsia B) Fatigue C) Oppositional defiant disorder D) Depression

A) Hemianopsia You should suspect a visual problem because there is no writing on one half of the circle. This is consistent with a hemianopsia, sometimes seen in stroke. These patients may also eat food on only one half of their plate. The other conditions would not account for this pattern.

You have just asked a patient how he feels about his emphysema. He becomes silent, folds his arms across his chest and leans back in his chair, and then replies, "It is what it is." How should you respond? A) "You seem bothered by this question." B) "Next, I would like to talk with you about your smoking habit." C) "Okay, let's move on to your other problems." D) "You have adopted a practical attitude toward your problem."

A) "You seem bothered by this question." You have astutely noted that the patient's body language changed at the time you asked this question, and despite the patient's response, you suspect there is more beneath the surface. Maybe he is afraid of being browbeaten about his smoking, maybe a relative has recently died from this disorder, or maybe a friend told him 20 years ago that he would eventually get emphysema. Regardless, by sharing your observation and leaving a pause, he may begin to talk about some issues which are very important to him.

A 30-year-old sales clerk comes to your office wanting to lose weight; her BMI is 30.02kg/m . What is the most appropriate amount for a weekly weight reduction goal? A) .5 to 1 pound per week B) 1 to 2.5 pounds per week C) 2.5 to 3.5 pounds per week D) 3.5 to 4.5 pounds per week

A) .5 to 1 pound per week Based on the NIH Obesity Guidelines, this is the weekly weight loss goal to strive for to maintain long-term control of weight. More rapid weight loss than this does not result in a better outcome at one year.

A woman in her third trimester complains of shortness of breath on occasion, without other symptoms. She has a normal examination. The most likely cause of this symptom is: A) Hormonal B) Asthma C) Pulmonary embolus D) Infection

A) Hormonal While these other etiologies must be considered in any dyspneic pregnant woman, the lack of other symptoms and findings makes it more likely that this is a hormonal effect of progesterone. Expect a normal respiratory rate and respiratory alkalosis.

During the delivery of a male infant, you are there to assess the Apgar score. He was born through an intact pelvis and had no complications during labor or delivery. At 1 minute he is pink all over and grimaces. He is flexing his arms and legs occasionally. He is breathing well and his heart rate is 110. At 5 minutes he is still pink all over but now is crying vigorously, with active movement. His respiratory effort is good and his heart rate is 130.What is his Apgar score? A) 8 at 1 minute, 10 at 5 minutes B) 7 at 1 minute, 9 at 5 minutes C) 9 at 1 minute, 10 at 5 minutes D) 8 at 1 minute, 9 at 5 minutes

A) 8 at 1 minute, 10 at 5 minutes In checking the Apgar, five things are looked at during the 1-minute and 5-minute marks. The color, reflex irritability, muscle tone, respiratory effort, and heart rate are evaluated. In this case, at 1 minute he received 2 points for being pink all over, 1 point for grimacing, 1 point for flexion of the arms and legs, 2 points for strong respiratory effort, and 2 points for a heart rate over 100. This gives a 1-minute total of 8. At 5 minutes he was given 2 points for being pink all over, 2 points for vigorous crying, 2 points for active movement, 2 points for strong breathing, and 2 points for a heart rate over 100. This gives a 5-minute total of 10. These are normal, healthy Apgar scores.

Which of the following questions is part of the screening for physical disability? A) Are you able to go shopping for groceries or clothes? B) Are you able to walk one block? C) Are you able to pass the driver's license test? D) Are you able to perform light dusting and pick up after yourself around the house?

A) Are you able to go shopping for groceries or clothes? This is part of the Physical Disability screening portion of the 10-Minute Geriatric Screener.

A 23-year-old graduate student comes to your clinic for evaluation of a urethral discharge. As the provider, you need to get a sexual history. Which one of the following questions is inappropriate for eliciting the information? A) Are you sexually active? B) When was the last time you had intimate physical contact with someone, and did thatcontact include sexual intercourse? C) Do you have sex with men, women, or both? D) How many sexual partners have you had in the last 6 months?

A) Are you sexually active? This is inappropriate because it is too vague. Given the complaint, you should probably assume that he is sexually active. Sometimes patients may respond to this question with the phrase "No, I just lie there." A specific sexual history will help you to assess this patient's risk for other sexually transmitted infections.

When you enter your patient's examination room, his wife is waiting there with him. Which of the following is most appropriate? A) Ask if it's okay to carry out the visit with both people in the room. B) Carry on as you would ordinarily. The permission is implied because his wife is in theroom with him. C) Ask his wife to leave the room for reasons of confidentiality. D) First ask his wife what she thinks is going on.

A) Ask if it's okay to carry out the visit with both people in the room. Even in situations involving people very familiar with each other, it is important to respect individual privacy. There is no implicit consent merely because he has allowed his wife to be in the room with him. On the other hand, it is inappropriate to assume that his wife should leave the room. Remember, the patient is the focus of the visit, so it would be appropriate to allow him to control who is in the room with him and inappropriate to address his wife first. Although your duty is to the patient, you may get optimal information by offering to speak to both people confidentially. This situation is analogous to an adolescent's visit.

Mrs. H. comes to your clinic, wanting antibiotics for a sinus infection. When you enter the room, she appears to be very angry. She has a raised tone of voice and states that she has been waiting for the past hour and has to get back to work. She states that she is unimpressed by the reception staff, the nurse, and the clinic in general and wants to know why the office wouldn't call in an antibiotic for her. Which of the following techniques is not useful in helping to calm this patient? A) Avoiding admission that you had a part in provoking her anger because you were late B) Accepting angry feelings from the patient and trying not to get angry in return C) Staying calm D) Keeping your posture relaxed

A) Avoiding admission that you had a part in provoking her anger because you were late In this scenario, the provider was 1 hour late in seeing the patient. The provider should acknowledge that he was late and apologize for this, no matter the reason for being late. It often helps to acknowledge that a patient's anger with you is understandable and that you might be angry in a similar situation.

A 55-year-old bookkeeper comes to your office for a routine visit. You note that on a previous visit for treatment of contact dermatitis, her blood pressure was elevated. She does not have prior elevated readings and her family history is negative for hypertension. You measure her blood pressure in your office today. Which of the following factors can result in a false high reading? A) Blood pressure cuff is tightly fitted. B) Patient is seated quietly for 10 minutes prior to measurement. C) Blood pressure is measured on a bare arm. D) Patient's arm is resting, supported by your arm at her mid-chest level as you stand tomeasure the blood pressure.

A) Blood pressure cuff is tightly fitted. A blood pressure cuff that is too tightly fitted can result in a false high reading. The other answers are important to observe to obtain an accurate blood pressure reading. JNC-7 also mentions the importance of having the back supported when obtaining blood pressure in the sitting position.

A 26-year-old telephone operator comes to your office for her first prenatal visit. This is her first pregnancy. Her last period was about 2 months ago. She has no current complaints. She is eating healthily, taking vitamins, and exercising. She has a past medical history of an appendectomy as a teenager. Her mother had three children vaginally with no complications. On examination she appears healthy and her vital signs are unremarkable. Her head, eyes, ears, nose, throat, thyroid, cardiac, pulmonary, and abdominal examinations are also unremarkable. By speculum examination, her cervix appears bluish in color and highly vascular. A bimanual examination reveals a soft cervix and a 12-week-sized uterus. No masses are felt in either adnexal area. Results of her Pap, cultures, and blood work are pending. What clinical sign is responsible for her blue, highly vascular cervix? A) Chadwick's sign B) Hegar's sign C) Leopold's sign D) Leo's sign

A) Chadwick's sign Chadwick's sign is observed during the speculum examination when the cervix appears more vascular and takes on a bluish hue. It can also occur with ectopic pregnancy.

Which of the following represents age-related changes in the lungs? A) Decrease in chest wall compliance B) Speed of expiration increases C) Increase in respiratory muscle strength D) Increased elastic recoil of lung tissue

A) Decrease in chest wall compliance The lungs age along with the rest of the body. These changes include decreased lung and chest wall compliance, increased expiratory time, decreased muscle strength and cough, and decreased elastic recoil.

Which of the following will help to optimize yield from a pediatric examination? A) Doing the examination out of order if necessary to take advantage of quiet periods for auscultation, etc. B) Being very orderly, so as not to miss a portion of the examination C) Using firmness as needed to make it through your examination D) Making sure to place the infant on the table during the examination while mom watchesclose by

A) Doing the examination out of order if necessary to take advantage of quiet periods for auscultation, etc. While order and routine are comforting to the examiner, children should be examined in an order which allows maximum yield. Many prefer to listen to the heart and lungs first while the child is quiet, in a parent's arms. Likewise, you may gain advantage to examining the mouth while the baby is crying. Most view the ENT examination as the most invasive for a child (especially the otoscopic examination), so many leave this for last.

An adolescent male comes to your clinic with a note from his mother stating it is okay for him to be seen today without her presence. He has come in for his annual sports physical required to play football. For his age his physical examination is unremarkable and you sign his school's physical examination form. You decide to take this opportunity to do some health education with him. He admits to wondering a lot lately if he is normal. Although he is in football he really enjoys science and computers more. He is worried that all his buddies will think he is a geek. He is convinced he also won't get a date for the Sadie Hawkins dance next week because the girls all think he is boring, too. He denies any experimentation with tobacco or alcohol, and he blushes when you mention sex. After hitting all the pertinent age-appropriate education points you give him his sports physical form and he leaves. The patient's concerns during the visit most resemble what developmental stage of adolescence? A) Early adolescence (10 to 14 years old) B) Middle adolescence (15 to 16 years old) C) Late adolescence (17 to 20 years old)

A) Early adolescence (10 to 14 years old) His concern with whether he is normal or not is often seen in the development of social identity in early adolescence. He is also concerned with the present (Sadie Hawkins dance) and not the distant future, as is seen with late adolescence. He also denies the experimentation often seen in middle adolescence.

A foster mother brings a 4-year-old child to see you for an evaluation. She has had custody of the girl for 2 weeks. She knows that the child was born in your state and that her maternal grandmother had custody for 6 months. She received good medical care during that time, but after her biologic mother obtained custody the child was abused and has had no further medical care. She says the child has had many behavioral problems and seems to be very behind on her developmental tasks. When you examine the child you notice short palpebral fissures, a wide nasal philtrum, and thin lips. Her cardiac, pulmonary, musculoskeletal, and abdominal examinations are normal. Her Denver Developmental Screening Test shows most of her milestones have occurred only through the 24th month. What form of congenital retardation is she most likely to have? A) Fetal alcohol syndrome B) Congenital hypothyroidism C) Down syndrome

A) Fetal alcohol syndrome The facial appearance in fetal alcohol syndrome shows short palpebral fissures, a wide and flattened philtrum, and thin lips. These children often have mild retardation even with good care, but with abuse they may have more profound retardation. This condition may occur with only modest alcohol consumption.

Which of the following is true of assessment of the vascular system in the elderly? A) Fewer than one third of patients with peripheral vascular disease have symptoms of claudication. B) An aortic width of 2.5 cm is abnormal. C) Bruits are commonly benign findings. D) Orthostatic blood pressure and pulse are not useful in this population.

A) Fewer than one third of patients with peripheral vascular disease have symptoms of claudication. It is the minority of patients with peripheral vascular disease who experience claudication; therefore, ankle-brachial ratios should be performed more frequently. The aorta should be 3 cm or less. Bruits usually indicate pathology, and even when there is not a significant blockage, the risk of vascular disease throughout the body is increased. Orthostatic vital signs are very useful in this population. Remember to observe the pulse as well, as failure of the heart to increase its rate is a common cause of orthostatic hypotension. This can occur as a result of autonomic neuropathy or medications such as beta-blockers, among other causes.

You are having trouble examining the abdomen of a school-aged child due to ticklishness. What should you do? A) Have the child press on your hand. B) Have the parent insist that the child allow you to examine her. C) Ask the parent to leave the room. D) Make the child realize that this is part of the examination and must be done.

A) Have the child press on your hand. By having the child participate in the examination and pressing on your hand, it will eliminate the ticklishness. Resistance to examination at this age is normal. The last three options only make the situation worse. The key is to have the child participate in the examination in a fun way.

A 27-year-old woman is brought to your office by her mother. The mother tells you that her daughter has been schizophrenic for the last 8 years and is starting to decompensate despite medication. The patient states that she has been taking her antipsychotic and she is doing just fine. Her mother retorts that her daughter has become quite paranoid. When asked why, the mother gives an example about the mailman. She says that her daughter goes and gets the mail every day and then microwaves the letters. The patient agrees that she does this but only because she sees the mailman flipping through the envelopes and she knows he's putting anthrax on the letters. Her mother turns to her and says, "He's only sorting the mail!" Which best describes the patient's abnormality of perception? A) Illusion B) Hallucination C) Fugue state

A) Illusion An illusion is merely a misinterpretation of real external stimuli. In this case, the mailman is looking through the letters before he puts them in the box. The mother correctly assumes he is sorting the mail but her schizophrenic daughter attributes his actions to being part of a nefarious bioterrorism plot.

Which of the following is the major effect of placental hormones? A) Insulin resistance B) Increased tidal volume C) Relative hypercortisolism, which may trigger labor D) Decreased lower esophageal sphincter tone

A) Insulin resistance Placental hormones are thought to contribute to insulin resistance. Progesterone increases tidal volume and minute volume. Decreased lower esophageal sphincter tone is due to progesterone and estradiol. Near term, increases in placental CRH and ACTH produce a relative hypercortisolism which may trigger labor.

You are seeing an older patient who has not had medical care for many years. Her vital signs taken by your office staff are: T 37.2, HR 78, BP 118/92, and RR 14, and she denies pain. You notice that she has some hypertensive changes in her retinas and you find mild proteinuria on a urine test in your office. You expected the BP to be higher. She is not on any medications. What do you think is causing this BP reading, which doesn't correlate with the other findings? A) It is caused by an "auscultatory gap." B) It is caused by a cuff size error. C) It is caused by the patient's emotional state. D) It is caused by resolution of the process which caused her retinopathy and kidney problems.

A) It is caused by an "auscultatory gap." The blood pressure is unusual in this case in that the systolic pressure is normal while the diastolic pressure is elevated. Especially with the retinal and urinary findings, you should consider that the BP may be much higher and that an auscultatory gap was missed. This can be avoided by checking for obliteration of the radial pulse while the cuff is inflated. Although a large cuff can cause a slightly lower BP on a patient with a small arm, this does not account for the elevated DBP. Emotional upset usually causes elevation of the BP. Although a process which caused the retinopathy and kidney problems may have resolved, leaving these findings, it is a dangerous assumption that this is the sole cause of the problems seen in this patient.

An 18-year-old college freshman presents to the clinic for evaluation of gastroenteritis. You measure the patient's temperature and it is 104 degrees Fahrenheit. What type of pulse would you expect to feel during his initial examination? A) Large amplitude, forceful B) Small amplitude, weak C) Normal D) Bigeminal

A) Large amplitude, forceful Fever results in an increased stroke volume, which results in a large-amplitude, forceful pulse. Later in the course of the illness, if dehydration and shock result, you may expect small amplitude and weak pulses.

A 72-year-old retired truck driver comes to the clinic with his wife for evaluation of hearing loss. He has noticed some decreased ability to hear what his wife and grandchildren are saying to him. He admits to lip-reading more. He has a history of noise exposure in his young adult years: He worked as a sound engineer at a local arena and had to attend a lot of concerts. Based on this information, what is the most likely finding regarding his hearing acuity? A) Loss of acuity for middle-range sounds B) Increase of acuity for low-range sounds C) Loss of acuity for high-range sounds D) Increase of acuity for high-range sounds

A) Loss of acuity for middle-range sounds Human speech is considered to be a middle-range sound. During the aging process there is a loss of acuity, starting with high-pitched sounds but extending to the middle range and then into the low range.

During cardiac examination you notice a new parasternal systolic murmur of 2/6 intensity. On palpation, the PMI is slightly higher than usual. What do you suspect? A) Mammary souffle B) Mitral stenosis C) Mitral regurgitation D) Aortic insufficiency

A) Mammary souffle Although a mammary souffle may be diastolic, the murmurs of mitral stenosis and aortic insufficiency cannot be heard in systole. Mitral regurgitation is a possibility, but the upward shift in PMI and lack of other symptoms make this less likely. Any new diastolic murmur should be investigated further.

A young mother presents with a pregnancy confirmed by urine HCG. Her LMP was June 20. Using Naegele's rule, you estimate what day of delivery? A) March 27 B) April 13 C) February 20 D) February 13

A) March 27 Naegele's rule calculates the EDD by adding 7 days and subtracting 3 months from the date of the last period.

A 20-year-old college student comes in with symptoms of fatigue, nausea, and an increase in urination. Her last period was 3 months ago (June 20, 2008). She is sexually active and always uses condoms. Her past medical history is unremarkable. On examination you see a young, anxious-appearing woman. Her vital signs are unremarkable. Her head, eyes, ears, throat, neck, thyroid, cardiac, pulmonary, and abdominal examinations are unremarkable. On pelvic examination a soft cervix is palpated and a 14-week-sized uterus is palpated. A urine pregnancy test is positive. You then inform the patient that she is expecting and, using Naegele's rule, give her the estimated date of confinement (EDC, or due date).What was the due date you gave her? A) March 27, 2009 B) March 13, 2009 C) September 27, 2009D) March 20, 2009

A) March 27, 2009 By Naegele's rule you add 7 days to the last menstrual period and then subtract 3 months. This gives you March 27, 2009.

Mrs. Buckley is a 75-year-old widow who wants you to look at her teeth because over the past 2 weeks she has had right-sided jaw pain when eating. It does not occur otherwise. She also has had a headache. Which of the following should be considered? A) Palpation of her temples B) Dental referral C) Ultrasound of the gallbladder D) Inquiry about anosmia

A) Palpation of her temples This story can be consistent with temporal arteritis, which can cause blindness in 15% of those affected. Early recognition is crucial. Most of these patients will have tenderness over one or both of the temporal arteries, and some have diminished temporal pulses as well. Early treatment with corticosteroids is indicated. It can also be associated with polymyalgia rheumatic, a condition which causes pain in the shoulder girdles and pelvis.

Jason is a 41-year-old electrician who presents to the clinic for evaluation of shortness of breath. The shortness of breath occurs with exertion and improves with rest. It has been going on for several months and initially occurred only a couple of times a day with strenuous exertion; however, it has started to occur with minimal exertion and is happening more than a dozen times per day. The shortness of breath lasts for less than 5 minutes at a time. He has no cough, chest pressure, chest pain, swelling in his feet, palpitations, orthopnea, or paroxysmal nocturnal dyspnea.Which of the following symptom attributes was not addressed in this description? A) Severity B) Setting in which the symptom occurs C) Timing D) Associated manifestations

A) Severity The severity of the symptom was not recorded by the interviewer, so we have no understanding as to how bad the symptom is for this patient. The patient could have been asked to rate his pain on a 0 to 10 scale or used one of the other standardized pain scales available.This allows the comparison of pain intensity before and after an intervention.

A 38-week gestation, 2500-gram infant is placed on your service. How would she be described? A) Term, normal birth weight B) Term, low birth weight C) Preterm, normal birth weight D) Preterm, low birth weight

A) Term, normal birth weight Preterm is defined as less than 37 weeks; term, 37-42 weeks; and post-term, over 42 weeks of gestation. Birth weights have similar limits: extremely low birth weight, less than 1000 grams; very low birth weight, less than 1500 grams; low birth weight, less than 2500 grams; and normal birth weight, equal to or more than 2500 grams. These have prognostic implications and impact on how closely to watch and how aggressively to treat these infants.

Which of the following booster immunizations is recommended in the older adult population? A) Tetanus B) Diphtheria C) Measles D) Mumps

A) Tetanus Older adults who have received the primary series of three tetanus immunizations should receive the single booster dose of tetanus immunization every 10 years.

Which of the following brief screening measures is useful in assessing memory? A) Three-item recall B) Serial 7s C) Spelling "world" backward D) Copying intersecting pentagrams

A) Three-item recall If the patient is unable to remember three items after 1 minute has passed, then this is a positive screening test and indicates a need for further testing. This is part of the "10-Minute Geriatric Screener."

A mother brings her 15-month-old daughter to your office for evaluation of a rash and fever. She says the rash started one day and the fever developed the next day. Her daughter has had all of her vaccinations up to 10 months. The mother sheepishly admits that she hasn't had time to bring her daughter in since her 10-month check-up. On examination you see a mildly sick-appearing toddler with a 102-degree temperature. Looking at her skin you see at least 100 of a variety of papules, vesicles, and ulcers in different stages of development. What illness prevented by proper vaccination does this toddler have? A) Varicella (chickenpox) B) Measles C) Smallpox

A) Varicella (chickenpox) Normally the first vaccine for varicella is given at 12 to 15 months. The characteristic rash in waves of lesions is in a pattern of papules, vesicles, ulcers, and scabs. Because of the number of persons who still get shingles (an outbreak of varicella in one dermatome following the original infection by years), there is still enough virus in the United States to easily get chickenpox without vaccination. This child is regarded as contagious to others until all of the lesions are "scabbed over." Smallpox would appear different in that all of the lesions would be in the same stage of development.

A 75-year-old homemaker brings her 76-year-old husband to your clinic. She states that 4 months ago he had a stroke and ever since she has been frustrated with his problems with communication. They were at a restaurant after church one Sunday when he suddenly became quiet. When she realized something was wrong he was taken to the hospital by EMS. He spent 2 weeks in the hospital with right-sided weakness and difficulty speaking. After hospitalization he was in a rehab center, where he regained the ability to walk and most of the use of his right hand. He also began to speak more, but she says that much of the time "he doesn't make any sense." She gives an example that when she reminded him the car needed to be serviced he told her "I will change the Kool-Aid out of the sink myself with the ludrip." She says that these sayings are becoming frustrating. She wants you to tell her what is wrong and what you can do about it. While you write up a consult to neurology, you describe the syndrome to her.What type of aphasia does he have? A) Wernicke's aphasia B) Broca's aphasia C) Dysarthria

A) Wernicke's aphasia With Wernicke's aphasia the patient can speak effortlessly and fluently, but his words often make no sense. Words can be malformed or completely invented. Wernicke's area is found on the temporal lobes.

A woman in her 30th week has a cervical length estimated at 1 cm. Should you be concerned? A) Yes; she may be at risk for preterm labor. B) Yes; she most likely has a bicornuate uterus. C) No; this is a normal measurement for this gestational age. D) Yes; it likely indicates the fetus is in the breech position.

A) Yes; she may be at risk for preterm labor. A normal cervical length prior to 34 to 36 weeks' gestation is 1.5 to 2 cm. Shorter lengths indicate the patient may be at risk for preterm labor. Cervical lengths do not correlate with the other choices.

You are examining for fetal heart tones with a fetoscope and are unable to hear any.Using a Doptone, you measure the rate as 164. Which gestational age is most likely? A) 8 weeks B) 14 weeks C) 20 weeks D) 26 weeks

B) 14 weeks With a fetoscope, the fetal heart should be audible starting at about 18 weeks. The Doptone is able to detect this at about 10 weeks. This would put the gestational age at between 10 and 18 weeks.

Mrs. Kelly comes to you for her usual prenatal check-up. You measure the fundal height at 24 cm. What would you estimate the length of her gestation to be? A) 20 weeks B) 24 weeks C) 28 weeks D) 32 weeks

B) 24 weeks The fundal height is an approximation of the number of weeks of gestation. For a more accurate estimate, see the figure at the top of page 883.

A toddler is able to jump in place and balance on one foot as well. She can also speak infull sentences and feed herself. What is the approximate developmental age of this child? A) 2 years B) 3 years C) 4 years D) 5 years

B) 3 years These milestones are consistent with a physical, cognitive/language, and social and emotional developmental age of 3 years.

A 32-year-old white female presents to labor and delivery fully effaced and delivers a 5.8-lb (2,500-gram) infant female with Apgar scores of 6 and 8. The mother has had no prenatal care and in the nursery you perform the newborn examination. With the Ballard scoring system, the neuromuscular examination score is 15. Looking at physical maturity, you see superficial peeling and few veins on the skin. The lanugo hair has bald areas and the plantar surface of the foot has creases on two thirds of it. The areola is stippled with a 2-mm bud. The pinna is well curved, is firm, and has instant recoil. The labia majora and minora are equally prominent. Add the score of the neuromuscular components to your score of physical maturity to determine weeks of gestation. How many weeks of gestation has this child had? A) 34 weeks B) 36 weeks C) 40 weeks

B) 36 weeks Superficial peeling with few veins gives a score of 2 points; lanugo with balding areas gives a score of 3 points; the plantar surface being covered by two thirds gives a score of 3 points; the stippled areola with a 2-mm bud gives a score of 2 points; the well-formed pinna with instant recoil gives a score of 3 points. The equal labia majora and labia minora give a score of 2 points. Adding these numbers up gives a score of 15 points for physical maturity. Adding that to the 15 points for neuromuscular maturity gives a point total of 30, which correlates to a gestational age of 36 weeks. This would be expected with a birth weight of 2,500 grams.

Mr. White's son brings him in today because he notes that Mr. White has not been himself lately. He seems forgetful and has not taken care of himself as he normally does. He has reported falling twice at home to his son and has telephoned late at night because of insomnia. His blood pressure and diabetes have been difficult to control and his warfarin dosing has become more difficult. Which of the following should you suspect? A) Alzheimer's dementia B) Alcohol use C) Urinary tract infection D) Stroke

B) Alcohol use All of these answers are common diseases of the elderly and many have atypical presentations in this age group. The fact that his hypertension has become more difficult to control and his warfarin dosing is challenging to manage should lead you to consider that there is alcohol use. Further questioning, quantifying his use of alcohol, and application of the CAGE questionnaire may be useful.

You are going to obtain a social history on an early adolescent boy. How should you proceed to obtain the best information? A) Ask his mother to leave the room. B) Ask if he would prefer his mother to leave the room. C) Ask your questions with his mother in the room. D) Ask his mother how she would like to proceed.

B) Ask if he would prefer his mother to leave the room. It is best to ask the patient what he or she would prefer. Because the examination should include a genitalia examination, some children in early adolescence are more comfortable with their parents in the room. Some examiners will provide "confidential time" to both the adolescent and the parent, so that parental concerns can also be adequately addressed. Leaving the parent in the room without asking the adolescent is usually not a good idea and can limit optimal history gathering and examination.

Alexandra is a 28-year-old editor who presents to the clinic with abdominal pain. The pain is a dull ache, located in the right upper quadrant, that she rates as a 3 at the least and an 8 at the worst. The pain started a few weeks ago, it lasts for 2 to 3 hours at a time, it comes and goes, and it seems to be worse a couple of hours after eating. She has noticed that it starts after eating greasy foods, so she has cut down on these as much as she can. Initially it occurred once a week, but now it is occurring every other day. Nothing makes it better. From this description, which of the seven attributes of a symptom has been omitted? A) Setting in which the symptom occurs B) Associated manifestations C) Quality D) Timing

B) Associated manifestations The interviewer has not recorded whether or not the pain has been accompanied by nausea, vomiting, fever, chills, weight loss, and so on. Associated manifestations are additional symptoms that may accompany the initial chief complaint and that help the examiner to start refining his or her differential diagnosis.

A 26-year-old white female comes to your clinic at 38 weeks, complaining of intermittent contractions. They last for 30 seconds and are coming every 10 minutes. Her prenatal course has so far been uneventful. You send her to labor and delivery for a labor assessment. On vaginal examination she has effaced 4 cm, but you cannot feel a presenting part. You admit her for active labor; however, you wish to assess if she is vertex (baby's head is down), so you do the Leopold's maneuver. Palpating the upper pole with your hands, you feel a firm round mass. Placing your hand along the right side of her abdomen, you feel a smooth firmness. Palpating your other hand along the left side of her abdomen, you feel irregular bumps. Above the pelvic brim you feel a firm irregular mass. While awaiting ultrasound to confirm your diagnosis, you write the pertinent orders. How is this fetus presenting? A) Vertex B) Breech C) Transverse

B) Breech When a baby is breech you often cannot feel a presenting part with the cervical check. In a breech lie, the irregular firm mass (the buttock) will be at the pelvis, while the firm round mass (the head) will be at the uterine pole.

Mrs. T. comes for her regular visit to the clinic. She is on your schedule because her regular provider is on vacation and she wanted to be seen. You have heard about her many times from your colleague and are aware that she is a very talkative person. Which of the following is a helpful technique to improve the quality of the interview for both the provider and the patient? A) Allow the patient to speak uninterrupted for the duration of the appointment. B) Briefly summarize what you heard from the patient in the first 5 minutes and then try to have her focus on one aspect of what she told you. C) Set the time limit at the beginning of the interview and stick with it, no matter what occurs in the course of the interview. D) Allow your impatience to show so that the patient picks up on your nonverbal cue that the appointment needs to end.

B) Briefly summarize what you heard from the patient in the first 5 minutes and then try to have her focus on one aspect of what she told you. You can also say, "I want to make sure I take good care of this problem because it is very important. We may need to talk about the others at the next appointment. Is that okay with you?" This is a technique that can help you to change the subject but, at the same time, validate the patient's concerns; it also can provide more structure to the interview.

A 72-year-old African-American male is brought to your clinic by his daughter for a follow-up visit after his recent hospitalization. He had been admitted to the local hospital for speech problems and weakness in his right arm and leg. On admission his MRI showed a small stroke. The patient was in rehab for 1 month following his initial presentation. He is now walking with a walker and has good use of his arm. His daughter complains, however, that everyone is still having trouble communicating with the patient. You ask the patient how he thinks he is doing. Although it is hard for you to make out his words you believe his answer is "well . . . fine . . . doing . . . okay." His prior medical history involved high blood pressure and coronary artery disease. He is a widower and retired handyman. He has three children who are healthy. He denies tobacco, alcohol, or drug use. He has no other current symptoms. On examination he is in no acute distress but does seem embarrassed when it takes him so long to answer. His blood pressure is 150/90 and his other vital signs are normal. Other than his weak right arm and leg his physical examination is unremarkable.What disorder of speech does he have? A) Wernicke's aphasia B) Broca's aphasia C) Dysarthria

B) Broca's aphasia In Broca's aphasia patients articulate very slowly and with a great deal of effort. Nouns, verbs, and important adjectives are usually present and only small grammatical words are dropped from speech. Broca's area is on the lateral portion of the frontal lobes.

You are observing a patient with heart failure and notice that there are pauses in his breathing. On closer examination, you notice that after the pauses the patient takes progressively deeper breaths and then progressively shallower breaths, which are followed by another apneic spell. The patient is not in any distress. You make the diagnosis of: A) Ataxic (Biot's) breathing B) Cheyne-Stokes respiration C) Kussmaul's respiration D) COPD with prolonged expiration

B) Cheyne-Stokes respiration Cheyne-Stokes respiration can be seen in patients with heart failure and is usually not a sign of an immediate problem. Ataxic breathing is very irregular in rhythm and depth and is seen with brain injury. Kussmaul's respiration is seen in patients with a metabolic acidosis, as they are trying to rid their bodies of carbon dioxide to compensate. Respirations in COPD are usually regular and are not usually associated with apneic episodes.

A 73-year-old retired accountant comes to your office for her annual examination. She has incontinence of urine when she coughs or sneezes. She takes several medications for control of hypertension and diabetes. You use the DIAPERS mnemonic to assess the cause of her incontinence. All of the following are items represented by the mnemonic except for: A) Atrophic vaginitis B) Depression C) Pharmaceuticals D) Restricted mobility

B) Depression Depression is not a risk factor for incontinence. The D in the mnemonic stands for delirium.

A 35-year-old stockbroker comes to your office, complaining of feeling tired and irritable. She also says she feels like nothing ever goes her way and that nothing good ever happens. When you ask her how long she has felt this way she laughs and says, "Since when have I not?" She relates that she has felt pessimistic about life in general since she was in high school. She denies any problems with sleep, appetite, or concentration, and states she hasn't thought about killing herself. She reports no recent illnesses or injuries. She is single. She smokes one pack of cigarettes a day, drinks occasionally, and hasn't taken any illegal drugs since college. Her mother suffers from depression and her father has high blood pressure. On examination her vital signs and physical examination are unremarkable.What mental health disorder best describes her symptoms? A) Major depressive episode B) Dysthymic disorder C) Cyclothymic disorder

B) Dysthymic disorder Someone with dysthymia has a depressed mood and symptoms for most of the day, more days than not, for at least 2 years. The disorder generally begins in adolescence and is fairly stable throughout life. Although the symptoms are similar to those of major depression (in this case, fatigue and irritability), they are milder and fewer.

A 24-year-old cashier comes to your clinic for her first OB visit. She had her last period 10 weeks before, which would mean she is 12 weeks pregnant. She did a home pregnancy urine test a month ago and it was positive. She has had some fatigue and nausea, but not in the last week. She has had no cramping or bleeding. Her vital signs, head, eyes, ears, nose, throat, thyroid, cardiac, pulmonary, and abdominal examinations are all unremarkable. On speculum examination her os is closed and there is a pinkish hue to the cervix. On bimanual examination the cervix is soft and the uterus is enlarged to the pelvic brim. Despite 20 minutes of trying, you cannot find heart tones. You repeat a urine pregnancy test and it is negative. A serum pregnancy test is ordered and is positive. You send the patient for a vaginal ultrasound. What is the most likely explanation for her presentation? A) Earlier than 12 weeks B) Fetal demise (missed abortion) C) False pregnancy

B) Fetal demise (missed abortion) This patient has lost some of the symptoms of pregnancy, including fatigue and nausea. Also, there are no fetal heart tones despite the 12-week uterus. A negative urine test indicated decreasing levels of beta HCG of pregnancy, although there is still enough to pick up on the more sensitive serum test.

A 29-year-old woman comes to your office. As you take the history, you notice that she is speaking very quickly, and jumping from topic to topic so rapidly that you have trouble following her. You are able to find some connections between ideas, but it is difficult. Which word describes this thought process? A) Derailment B) Flight of ideas C) Circumstantiality D) Incoherence

B) Flight of ideas This represents flight of ideas because the ideas are connected in some logical way. Derailment, or loosening of associations, has more disconnection within clauses. Circumstantiality is characterized by the patient speaking "around" the subject and using excessive detail, though thoughts are meaningfully connected. Incoherence lacks meaningful connection and often has odd grammar or word use. Although severe flight of ideas can produce this condition, evidence is not present in this vignette.

Jeannie is a 24-year-old pregnant woman who asks you today if her frequent urination is normal. Which of the following hormones is most likely responsible for this? A) TSH B) HCG C) Oxytocin D) Estradiol

B) HCG HCG resets the thirst receptors and releases ADH. Estradiol increases prolactin output in the anterior lobe of the pituitary to ready the breasts for lactation. Thyroid output increases in pregnancy, but the free T3 and T4 are kept relatively constant because of the increase in thyroid binding globulin.

A 22-year-old man is brought to your office by his father to discuss his son's mental health disorder. The patient was diagnosed with schizophrenia 6 months ago and has been taking medication since. The father states that his son's dose isn't high enough and you need to raise it. He states that his son has been hearing things that don't exist. You ask the young man what is going on and he tells you that his father is just jealous because his sister talks only to him. His father turns to him and says, "Son, you know your sister died 2 years ago!" His son replies "Well, she still talks to me in my head all the time!"Which best describes this patient's abnormality of perception? A) Illusion B) Hallucination C) Fugue state

B) Hallucination A hallucination is a subjective sensory perception in the absence of real external stimuli. The patient can hear, see, smell, taste, or feel something that does not exist in reality. In this case, his sister has passed away and cannot be speaking to him, although in his mind he can hear her. This is an example of an auditory hallucination, but hallucinations can occur with any of the five senses.

An infant presents with a heart rate of 180, a respiratory rate of 68, and an enlarged liver. What diagnosis does this suggest? A) Pneumonia B) Heart failure C) Sepsis D) Necrotizing enterocolitis

B) Heart failure Heart failure presents differently in infants than in adults. This triad should suggest this diagnosis. Pneumonia, necrotizing enterocolitis, and sepsis should not necessarily cause hepatomegaly. Observe closely for central cyanosis of the lips and tongue. Peripheral cyanosis alone does not mean much in infants. Perform a careful cardiac examination in as quiet a setting as possible, perhaps while the infant is in the mother's arms, to look for evidence of valvular disease.

A mother brings her 4-year-old daughter to your office because of fever and decreased eating and drinking. When you ask the little girl what is wrong, she says her mouth and throat hurt. On examination her temperature is 101 degrees. Her ears and nose examinations are unremarkable. Her mouth has ulcerations on the buccal mucosa and the tongue. She also has cervical lymphadenopathy. Her cardiac and pulmonary examinations are normal. She is up to date on her childhood vaccinations.What mouth abnormality does she most likely have? A) Strep throat B) Herpetic stomatitis C) Oral candidiasis (thrush) D) Diphtheria

B) Herpetic stomatitis With herpetic stomatitis there is often a low-grade fever with small ulcers covering the mucosa of the mouth. The pain from the ulcers leads to decreased oral intake and even dehydration.

A 78-year-old retired seamstress comes to the office for a routine check-up. You obtain an ECG (electrocardiogram) because of her history of hypertension. You diagnose a previous myocardial infarction and ask her if she had any symptoms related to this. Which of the following symptoms would be more common in this patient's age group for an acute myocardial infarction? A) Chest pain B) Syncope C) Pain radiating into the left arm D) Pain radiating into the jaw

B) Syncope This is an atypical symptom and more likely to be seen in this patient's age group.

Mr. Chin is an 82-year-old man who comes to your office for a routine check. On examination, you notice a somewhat high-pitched murmur in the second right intercostal space during systole. It does not radiate and the rest of his examination is normal for his age. Which is true of the most likely cause of this murmur? A) It often decreases carotid upstroke. B) It carries with it increased risk for cardiovascular disease. C) It is usually accompanied by an S3 gallop. D) It is found in 10% of otherwise normal elderly patients.

B) It carries with it increased risk for cardiovascular disease. This murmur most likely represents aortic sclerosis, a common murmur affecting about one third of those near 60 years of age. It is caused by calcification of the valve and is associated with cardiovascular risk. Aortic sclerosis does not usually cause obstruction to normal flow, so carotid upstroke should be normal, and it is not associated with an S3 gallop.

A 22-year-old clerk, primigravida, comes to your office for a prenatal visit. She is in her second trimester and has had prenatal care since she was 8 weeks pregnant. Her only complaint is that she has a new brownish line straight down her abdomen. On examination her vital signs are unremarkable. Her urine has no protein, glucose, or leukocytes. With a Doptone the fetal heart rate is 140, and her uterus is palpated to the umbilicus. Today you are sending her for congenital abnormality screening and setting up an ultrasound.What physical finding is responsible for her new "brown line"? A) Corpus luteum B) Linea nigra C) Linea alba D) Diastasis recti

B) Linea nigra The linea nigra is a linear hyperpigmented area of skin along the midline of the abdomen. It is caused by the hormonal changes of pregnancy. It is considered normal.

You have been unable to hear normal S2 splitting in children up to this point. What technique will maximize your chances of hearing this phenomenon? A) Listen with the diaphragm over the left lower sternal border. B) Listen with the bell over the 2nd left intercostal space. C) Listen with the bell over the apex. D) Listen with the diaphragm in the axilla.

B) Listen with the bell over the 2nd left intercostal space. S2 is made of aortic and pulmonic components. Of these, the pulmonic component is much softer and heard best over the pulmonic area. Even in the proper location, the pulmonic component may be difficult to hear with the diaphragm because it is a soft, low-pitched sound. For this reason, the bell should be used to listen for S2 splitting over the pulmonic area during inspiration, when splitting should be maximized. Breathing also changes heart rate more rapidly in children. One may think an arrhythmia is present until she notices that this rate change is related to the respiratory cycle.

A 23-year-old ticket agent is brought in by her husband because he is concerned about her recent behavior. He states that for the last 2 weeks she has been completely out of control. He says that she hasn't showered in days, stays awake most of the night cleaning their apartment, and has run up over $1,000 on their credit cards. While he is talking, the patient interrupts him frequently and declares this is all untrue and she has never been so happy and fulfilled in her whole life. She speaks very quickly, changing the subject often. After a longer than normal interview you find out she has had no recent illnesses or injuries. Her past medical history is unremarkable. Both her parents are healthy but the husband has heard rumors about an aunt with similar symptoms. She and her husband have no children. She smokes one pack of cigarettes a day (although she has been chain-smoking in the last 2 weeks), drinks four to six drinks a week, and smokes marijuana occasionally. On examination she is very loud and outspoken. Her physical examination is unremarkable.Which mood disorder does she most likely have? A) Major depressive episode B) Manic episode C) Dysthymic disorder

B) Manic episode Mania consists of a persistently elevated mood for at least 1 week with symptoms such as inflated self-esteem, decreased need for sleep, pressured speech, racing thoughts, and involvement in high-risk activities (such as drug use, spending sprees, and indiscriminate sexual activity). In this case, the patient has racing thoughts and pressured speech, has a decreased need for sleep, and is engaging in high-risk activities (spending sprees).

Mr. Garcia comes to your office for a rash on his chest associated with a burning pain. Even a light touch causes this burning sensation to worsen. On examination, you note a rashwith small blisters (vesicles) on a background of reddened skin. The rash overlies an entire rib on his right side. What type of pain is this? A) Idiopathic pain B) Neuropathic pain C) Nociceptive or somatic pain D) Psychogenic pain

B) Neuropathic pain This vignette is consistent with a diagnosis of herpes zoster, or shingles. This is caused by reemergence of dormant varicella (chickenpox) viruses from Mr. Garcia's nerve root. The characteristic burning quality without a history of an actual burn makes one think of neuropathic pain. It will most likely remain for months after the rash has resolved. There is no evidence of physical injury and this is a peculiar distribution, making nociceptive pain less likely. There is no evidence of a psychogenic etiology for this, and the presence of a rash makes this possibility less likely as well. Because of your astute diagnostic abilities, the pain is not idiopathic.

When using an interpreter to facilitate an interview, where should the interpreter be positioned? A) Behind you, the examiner, so that the lips of the patient and the patient's nonverbal cues can be seen B) Next to the patient, so the examiner can maintain eye contact and observe the nonverbal cues of the patient C) Between you and the patient so all parties can make the necessary observations D) In a corner of the room so as to provide minimal distraction to the interview

B) Next to the patient, so the examiner can maintain eye contact and observe the nonverbal cues of the patient Interpreters are invaluable in encounters where the examiner and patient do not speak the same language, including encounters with the deaf. It should be noted that deaf people from different regions of the world use different sign languages. The priority is for you to have a good view of the patient. Remember to use short, simple phrases while speaking directly to the patient and ask the patient to repeat back what he or she understands.

A 15-year-old high school sophomore and her mother come to your clinic because themother is concerned about her daughter's weight. You measure her daughter's height and weight and obtain a BMI of 19.5 kg/m . Based on this information, which of the following is appropriate? A) Refer the patient to a nutritionist and a psychologist because the patient is anorexic. B) Reassure the mother that this is a normal body weight. C) Give the patient information about exercise because the patient is obese. D) Give the patient information concerning reduction of fat and cholesterol in her diet becauseshe is obese.

B) Reassure the mother that this is a normal body weight. The patient has a normal BMI; the range for a normal BMI is 18.5 to 24.9 kg/m .You may be able to give the patient and her mother the lower limit of normal in pounds for her daughter's height, or instruct her in how to use a BMI table.

You are interviewing an elderly woman in the ambulatory setting and trying to get more information about her urinary symptoms. Which of the following techniques is not a component of adaptive questioning? A) Directed questioning: starting with the general and proceeding to the specific in a manner that does not make the patient give a yes/no answer B) Reassuring the patient that the urinary symptoms are benign and that she doesn't need to worry about it being a sign of cancer C) Offering the patient multiple choices in order to clarify the character of the urinary symptoms that she is experiencing D) Asking her to tell you exactly what she means when she states that she has a urinary tract infection

B) Reassuring the patient that the urinary symptoms are benign and that she doesn't need to worry about it being a sign of cancer Reassurance is not part of clarifying the patient's story; it is part of establishing rapport and empathizing with the patient.

A 49-year-old truck driver comes to the emergency room for shortness of breath and swelling in his ankles. He is diagnosed with congestive heart failure and admitted to the hospital. You are the student assigned to do the patient's complete history and physical examination. When you palpate the pulse, what do you expect to feel? A) Large amplitude, forceful B) Small amplitude, weak C) Normal D) Bigeminal

B) Small amplitude, weak Congestive heart failure is characterized by decreased stroke volume or increased peripheral vascular resistance, which would result in a small-amplitude, weak pulse. Subtle differences in amplitude are usually best detected in large arteries close to the heart, like the carotid pulse. You may not be able to notice these in other locations.

You are assessing an infant and notice that his nares flare, he has a soft grunt with each breath, and the skin between his ribs is pulled inward with inhalation. What is the significance of these findings? A) These are indicative of a CNS process. B) These are indicative of respiratory distress. C) These are indicative of muscular dystrophy. D) These are frequently accompanied by stridor.

B) These are indicative of respiratory distress. It is critical to notice these findings of respiratory distress. Muscular dystrophy may not allow the appearance of these signs because they are caused by muscular effort. It is hard to find a cause for these signs in the CNS. Stridor is usually inspiratory, so while nasal flaring and retractions may occur, grunting is unusual because exhalation is unimpeded.

Mr. Q. is a 45-year-old salesman who comes to your office for evaluation of fatigue. He has come to the office many times in the past with a variety of injuries, and you suspect that he has a problem with alcohol. Which one of the following questions will be most helpful in diagnosing this problem? A) You are an alcoholic, aren't you? B) When was your last drink? C) Do you drink 2 to 3 beers every weekend? D) Do you drink alcohol when you are supposed to be working?

B) When was your last drink? This is a good opening question that is general and neutral in tone; depending on the timing, you will be able to ask for more specific information related to the patient's last drink. The others will tend to stifle the conversation because they are closed-ended questions. Answer D implies negative behavior and may also keep the person from sharing freely with you.

A quiet 3-year-old is brought in for a routine check-up when you notice a fresh bruise in the axilla and bilateral bruises over the upper back that appear slightly older. There are brown bruises over his shins as well. His mother said this happened when he fell off of a couch. What diagnosis should be considered? A) Von Willebrand's disease B) Normal childhood bruises from activity C) Abuse D) Seizure disorder

C) Abuse No one wants to think that a child could be abused.shins are very normal for this age group with normal activity.areas which do not correlate with the given history are important to notice and should make you consider this diagnosis. A very thorough examination must be conducted to search for otherIn this case the bruises on the The presence of bruises in other lesions that might be consistent with the use of implements such as an electrical cord, clothes iron, cigarette, etc. A social services consult and/or formal abuse evaluation should be considered. Unfortunately, emotional and sexual abuse do not frequently leave outward signs. It is important to keep an open mind to the presence of these other types of abuse as well.

A 19-year-old childcare worker comes to you for her first prenatal visit. She cannot remember when her last period was but thinks it was between 2 and 5 months ago. When she began gaining weight and feeling "something" moving down there, she did a home pregnancy test and it was positive. She states she felt the movement about a week ago. She has had no nausea, vomiting, fatigue, or fevers. Her past medical history is remarkable only for irregular periods. She has been dating the same young man for a year. She says they were not using condoms. On examination you see an overweight young lady appearing her stated age. Her head, eyes, ears, nose, throat, neck, thyroid, cardiac, and pulmonary examinations are unremarkable. Her abdomen is nontender, with normal bowel sounds, and the gravid uterus is palpated to the level of the umbilicus. Fetal tones are easily found with Doptone, and with the fetoscope a faint heart rate of 140 is heard. By speculum examination the cervix is bluish and by bimanual examination the cervix is soft. Results of Pap smear, cultures, and blood work are pending. You give the patient her due date and how far along she is, based on your clinical findings. An OB ultrasound to confirm her dates is ordered.With only the clinical examination, how many weeks pregnant did you tell this patient she is? A) 6 to 8 weeks B) 12 to 14 weeks C) 18 to 20 weeks D) 24 to 26 weeks

C) 18 to 20 weeks Fetal tones can be easily found with Doptone and faintly auscultated with the fetoscope. The uterus is usually at the level of the umbilicus at 20 weeks. First-time mothers usually don't feel fetal movement until 19 to 20 weeks.

A 32-year-old attorney comes to your office for her second prenatal visit. She has had two previous pregnancies with uneventful prenatal care and vaginal deliveries. Her only problem was that with each pregnancy she gained 50 lbs (23 kg) and had difficulty losing the weight afterward. She has no complaints today. Looking at her chart, you see she is currently 10 weeks pregnant and that her prenatal weight was 130 lbs (59 kg). Her weight today is 134 lbs (60.9 kg). Her height is 5'4", giving her a BMI of 22. Her blood pressure, pulse, and urine tests are unremarkable. The fetal heart tone is difficult to find but is located and is 150. While you give her first trimester education, you tell her how much weight you expect her to gain.How much weight should this patient gain during pregnancy? A) Less than 15 pounds (less than 7 kg) B) 15 to 25 pounds (7 to 11.5 kg) C) 25 to 35 pounds (11.5 to 16 kg) D) 30 to 40 pounds (12.5 to 18 kg)

C) 25 to 35 pounds (11.5 to 16 kg) This is the appropriate amount of weight gain for a person with a normal BMI of 19.8 to 26.0.

You are observing an infant who is able to pull to a stand, uses "mama" and "dada" specifically, and indicates his wants by vocalization and pointing. Where would you place this child's developmental age? A) 12 months B) 10 months C) 8 months D) 6 months

C) 8 months Assessing developmental milestones is of major importance during the first year and beyond. These accomplishments in the physical, cognitive/language, and social domains are normal for an 8-month-old infant.

A patient comes to you for the appearance of red patches on his forearms that have been present for several months. They remain for several weeks. He denies a history of trauma. Which of the following is likely? A) Actinic keratoses B) Pseudoscars C) Actinic purpura D) Cherry angiomas

C) Actinic purpura Actinic purpura is a common benign skin condition of the elderly, frequently involving the forearms. Pseudoscars are white patches and cherry angiomas are bright-red raised lesions usually found on the torso. Actinic keratoses are lesions resembling nevi, often with features which would be concerning if considering melanoma (review the ABCDEs of melanoma), but they produce a slightly greasy scale when scratched with a nail.

On a very busy day in the office, Mrs. Donelan, who is 81 years old, comes for her usual visit for her blood pressure. She is on a low-dose diuretic chronically and denies any side effects. Her blood pressure is 118/78 today, which is well-controlled. As you are writing her script, she mentions that it is hard not having her husband Bill around anymore. What would you do next? A) Hand her the script and make sure she has a 3-month follow-up appointment. B) Make sure she understands the script. C) Ask why Bill is not there. D) Explain that you will have more time at the next visit to discuss this.

C) Ask why Bill is not there. Sometimes, the patient's greatest need is for support and empathy. It would be inappropriate to ignore this comment today. She may have relied heavily upon Bill for care and may be in danger. She may be depressed and even suicidal, but you will not know unless you discuss this with her. Most importantly, you should empathize with her by saying something like "It must be very difficult not to have him at home" and allow a pause for her to answer.You may also ask "What did you rely on him to do for you?" Only a life-threatening crisis with another patient should take you out of her room at this point, and you may need to adjust your office schedule to allow adequate time for her today.

Mrs. Lenzo weighs herself every day with a very accurate balance-type scale. She hasnoticed that over the past 2 days she has gained 4 pounds. How would you best explain this? A) Attribute this to some overeating at the holidays. B) Attribute this to wearing different clothing. C) Attribute this to body fluid. D) Attribute this to instrument inaccuracy.

C) Attribute this to body fluid. This amount of weight over a short period should make one think of body fluid changes. You may consider a kidney problem or heart failure in your differential. The other reasons should be considered as well, but this amount of weight gain over a short period usually indicates causes other than excessive caloric intake. A rule of thumb for dieters is that an energy excess of 3500 calories will cause a 1-pound weight gain, if the increase is to be attributed to food intake.

Common or concerning symptoms to inquire about in the General Survey and vital signs include all of the following except: A) Changes in weight B) Fatigue and weakness C) Cough D) Fever and chills

C) Cough This symptom is more appropriate to the respiratory review of systems.

It is summer and an 82-year-old woman is brought to you from her home after seeing her primary care doctor 2 days ago. She was started on an antibiotic at that time. Today, shecomes to the emergency room not knowing where she is or what year it is. likely cause of this? A) Alzheimer's dementia B) Stroke C) Delirium D) Meningitis

C) Delirium These are not signs of normal aging and seem to be of acute onset. This makes Alzheimer's less likely. Stroke and meningitis could cause these symptoms as well, but the combination of the heat and a recent infection make delirium much more likely. Though she was prescribed an antibiotic, she may not have improved because of bacterial resistance or because of noncompliance due to cost, depression, or even an underlying mild dementia. Dementia should not result in an acute mental status change, although illness may cause a worsening of dementia.

You are examining an elderly man and notice the following: decreased vibration sense in the feet and ankles, diminished gag reflex, right patellar reflex less than the left, and diminished abdominal reflexes. Which of these is abnormal? A) Decreased vibration sense B) Diminished gag reflex C) Diminished right patellar reflex compared to the left D) Diminished abdominal reflexes

C) Diminished right patellar reflex compared to the left Asymmetry on any examination is usually reason for concern. The other changes are commonly associated with aging. You may consider looking for other neurologic signs on the right, although occasionally you may mistake an abnormally brisk reflex to be normal when compared to the other side. It is usually a good idea to question whether the opposite side is actually the abnormal one when you find asymmetry on examination.

A pregnant woman finally comes in for her prenatal checkup. She complains today of headache and abdominal pain of several months' duration. She appears somewhat hurried or nervous. What question would you ask next? A) Do you have a family history of thyroid disease? B) Have you been eating properly and taking a prenatal vitamin? C) Do you feel safe at home? D) How much activity have you been able to fit into your schedule?

C) Do you feel safe at home? All of these questions are important in interviewing a pregnant woman. This picture may make you think of social problems such as domestic violence and/or substance abuse. Asking more directed questions in these areas may be fruitful.

Mrs. Geller is somewhat quiet today. She has several bruises of different colors on the ulnar aspects of her forearms and on her abdomen. She otherwise has no complaints and her diabetes and hypertension are well managed. Her son from out of state accompanies her today and has recently moved in to help her. What should you suspect? A) Overuse of aspirin B) Frequent falls C) Elder abuse D) Depression

C) Elder abuse The different colors of the bruising indicate that they have occurred at different times and are unlikely to have resulted from a single fall. The location of the bruising on the ulnar aspects of the forearms potentially indicates that she was trying to defend herself and are not typical areas to be bruised by a fall. Depression may be evident, but this is more likely to be a result rather than a cause of her situation today. While nothing is proven, it would be wise to interview her without her son in the room. If in doubt, a social worker consult may be helpful to determine if elder abuse is occurring.

Blood pressure abnormalities found more commonly in Western elderly include which of the following? A) Isolated elevation of the diastolic BP B) Narrow pulse pressure C) Elevation of the systolic BP D) Elevation of the BP with standing

C) Elevation of the systolic BP Isolated systolic hypertension is common in the elderly because of stiffening of the large arteries. This is often accompanied by widening of the pulse pressure. Orthostatic BP changes are often seen with postural changes and can account for falls as well.

A 79-year-old retired banker comes to your office for evaluation of difficulty with urination; he gets up five to six times per night to urinate and has to go at least that often in the daytime. He does not feel as if his bladder empties completely; the strength of the urinary stream is diminished. He denies dysuria or hematuria. This problem has been present for several years but has worsened over the last 8 months. You palpate his prostate. What is your expected physical examination finding, based on this description? A) Normal size, smooth B) Normal size, boggy C) Enlarged size, smooth D) Enlarged size, boggy

C) Enlarged size, smooth This is the expected physical examination finding in benign prostatic hyperplasia (BPH).

Mr. W. is a 51-year-old auto mechanic who comes to the emergency room wanting to be checked out for the symptom of chest pain. As you listen to him describe his symptom in more detail, you say "Go on," and later, "Mm-hmmm." This is an example of which of the following skilled interviewing techniques? A) Echoing B) Nonverbal communication C) Facilitation D) Empathic response

C) Facilitation This is an example of facilitation. Facilitation can be posture, actions, or words that encourage the patient to say more.

You are assessing Tanner staging of the breast in a young woman. You notice projectionof the areola and nipple to form a secondary mound above the level of the breast. Tanner stage would this be? A) I B) II C) III D) IV

C) III This would be a Tanner stage III because there is elevation of the nipple and areola above the level of the surrounding breast tissue and because the areola has not receded to the general contour of the breast.

A 35-year-old bus driver comes to your office for a prenatal visit. She is approximately 28 weeks pregnant and has had no complications. She is complaining only of heartburn and has had no fatigue, headaches, leg swelling, contractions, leakage of fluid, or bleeding. On examination her blood pressure is 142/92 and her urine shows no glucose, protein, or leukocytes. Her weight gain is appropriate, with no large recent increases. Fetal tones are 140 and her uterus measures 32 cm from the pubic bone. Looking back through her chart, you see her prenatal blood pressure was 120/70 and her blood pressures during the first 20 weeks were usually 120 to 130/70 to 80. What type of blood pressure is this? A) Normotensive for pregnancy B) Chronic hypertension C) Gestational hypertension D) Preeclampsia

C) Gestational hypertension Gestational hypertension occurs in women who are normotensive before pregnancy and develop systolic pressures over 140 and diastolic pressures over 90 after week 20 of pregnancy. There will be no protein in the urine and no symptoms of preeclampsia such as rapid weight gain, leg edema, or headaches. These patients must be cautioned about symptoms of preeclampsia and receive aggressive follow-up.

A 25-year-old radio announcer comes to the clinic for an annual examination. His BMI is226.0 kg/m . He is concerned about his weight. Based on this information, what is appropriatecounsel for the patient during the visit? A) Refer the patient to a nutritionist because he is anorexic. B) Reassure the patient that he has a normal body weight. C) Give the patient information about reduction of fat, cholesterol, and calories because he isoverweight. D) Give the patient information about reduction of fat and cholesterol because he is obese.

C) Give the patient information about reduction of fat, cholesterol, and calories because he isoverweight. The patient has a BMI in the overweight range, which is 25.0 to 29.9 kg/m. It is prudent to give him information about reducing calories, fat, and cholesterol in his diet to help prevent further weight gain.

Suzanne, a 25 year old, comes to your clinic to establish care. You are the student preparing to go into the examination room to interview her. Which of the following is the most logical sequence for the patient-provider interview? A) Establish the agenda, negotiate a plan, establish rapport, and invite the patient's story. B) Invite the patient's story, negotiate a plan, establish the agenda, and establish rapport. C) Greet the patient, establish rapport, invite the patient's story, establish the agenda, expandand clarify the patient's story, and negotiate a plan. D) Negotiate a plan, establish an agenda, invite the patient's story, and establish rapport.

C) Greet the patient, establish rapport, invite the patient's story, establish the agenda, expandand clarify the patient's story, and negotiate a plan. This is the most productive sequence for the interview. Greeting patients and establishing rapport allows them to feel more comfortable before "inviting" them to relate their story. After hearing the patient's story, together you establish the agenda regarding the most important items to expand upon. At the end, together you negotiate the plan of diagnosis and treatment.

A young woman comes to you with a cut on her finger caused by the lid of a can she was opening. She is pacing about the room, crying loudly, and through her sobs she says, "My career as a pianist is finished!" Which personality type exhibits these features? A) Narcissistic B) Paranoid C) Histrionic D) Avoidant

C) Histrionic The theatrical nature of her behavior as well as her overreaction lead to a diagnosis of histrionic character disorder.

A 29-year-old homemaker who is G4P3 comes to your clinic for her first prenatal check.Her last period was 2 months ago. She has had three previous pregnancies and deliveries with no complications. She has no medical problems and has had no surgeries. Her only current complaint is of severe reflux that occurs in the mornings and evenings. On examination she is in no acute distress. Her vitals are 110/70 with a pulse of 88. Her respirations are 16. Her head, eyes, ears, nose, throat, thyroid, cardiac, pulmonary, and abdominal examinations are unremarkable. On bimanual examination her cervix is soft and her uterus is 10 weeks in size. Pap smear, cultures, and blood work are pending.What is the most likely cause of her first-trimester reflux? A) Increasing prolactin levels B) Increasing ADH (antidiuretic hormone) levels C) Increasing progesterone D) Enlarged gravid uterus

C) Increasing progesterone Progesterone lowers the esophageal sphincter tone, leading to reflux and heartburn. It also relaxes tone and contraction of the ureters and bladder, increasing risk of UTI and subsequent bacteremia.

A mother brings her infant to you because of a "rattle" in his chest with breathing. Which of the following would you hear if there were a problem in the upper airway? A) Different sounds from the nose and chest B) Asymmetric sounds C) Inspiratory sounds D) Sounds louder in the lower chest

C) Inspiratory sounds It is important to distinguish upper airway sounds from lower because many benign conditions cause upper airway noise, such as viral upper respiratory infections. It is reassuring to hear the same noises at the nose as at the chest. Lower respiratory conditions also are generally symmetric, and sounds are louder at the upper chest versus the lower chest. They are usually very harsh and loud, which concerns parents.

Mrs. Stanton is a 79-year-old widow who presents to your office for a routine BP visit. You note a new pulsatile mass in the right neck at the carotid artery. Which of the following is the most likely cause for this? A) Anxiety B) Carotid artery aneurysm C) Kinking of the artery D) Tortuous aorta

C) Kinking of the artery While a carotid artery aneurysm is a possibility, it is more likely due to kinking of the carotid artery in this patient with HTN. A tortuous aorta will sometimes cause elevation of the left jugular vein by impairing drainage within the thorax.

A 32-year-old white female comes to your clinic, complaining of overwhelming sadness. She says for the past 2 months she has had crying episodes, difficulty sleeping, and problems with overeating. She says she used to go out with her friends from work but now she just wants to go home and be by herself. She also thinks that her work productivity has been dropping because she just is too tired to care or concentrate. She denies any feelings of guilt or any suicidal ideation. She states that she has never felt this way in the past. She denies any recent illness or injuries. Her past medical history consists of an appendectomy when she was a teenager; otherwise, she has been healthy. She is single and works as a clerk in a medical office. She denies tobacco, alcohol, or illegal drug use. Her mother has high blood pressure and her father has had a history of mental illness. On examination you see a woman appearing her stated age who seems quite sad. Her facial expression does not change while you talk to her and she makes little eye contact. She speaks so softly you cannot always understand her. Her thought processes and content seem unremarkable.What type of mood disorder do you think she has? A) Dysthymic disorder B) Manic (bipolar) disorder C) Major depressive episode

C) Major depressive episode Major depression occurs in a person with a previously normal state of mood. The symptoms often consist of a combination of sadness, decreased interest, sleeping problems (insomnia or hypersomnia), eating problems (decreased or increased appetite), feelings of guilt, decreased energy, decreased concentration, psychomotor changes (retardation or agitation), and a preoccupation with thoughts of death or suicide. There must be at least five symptoms for a diagnosis of major depression. This patient has six: (1) sadness, (2) trouble sleeping, (3) overeating, (4) fatigue, (5) difficulty with concentration, and (6) no interest in doing things.

A young Hispanic mother brings in her 2-month-old son. She is upset because her neighbors have threatened to call the Child Protective Agency because they think his birthmark is a bruise. Her son was the product of an uneventful pregnancy and spontaneous vaginal delivery. On examination you see a large, smooth-bordered bluish mark on his buttock and lower back. Otherwise his examination is unremarkable.What form of birthmark is this likely to be? A) Café-au-lait spot B) Salmon patch C) Mongolian spot

C) Mongolian spot Mongolian spots are large, smooth-bordered birthmarks found on the back and/or buttocks. They are found more often in darker-pigmented infants such as in the Hispanic or Asian population. They are commonly mistaken for bruises.

A 24-year-old secretary comes to your clinic, complaining of difficulty sleeping, severe nightmares, and irritability. She states it all began 6 months ago when she went to a fast food restaurant at midnight. While she was waiting in her car a man entered through the passenger door and put a gun to her head. He had her drive to a remote area, where he took her money and threatened to kill her. When the gun jammed he panicked and ran off. Ever since this occurred the patient has been having these symptoms. She states she jumps at every noise and refuses to drive at night. She states her anxiety has had such a marked influence on her job performance she is afraid she will be fired. She denies any recent illnesses or injuries. Her past medical history is unremarkable. On examination you find a nervous woman appearing her stated age. Her physical examination is unremarkable. You recommend medication and counseling.What anxiety disorder to you think this young woman has? A) Specific phobia B) Acute stress disorder C) Post-traumatic stress disorder D) Generalized anxiety disorder

C) Post-traumatic stress disorder Post-traumatic stress disorder is the fearful response (nightmares, avoidance of areas, irritability) to an event that occurred at least 1 month prior to presentation. The patient's fears and reactions cause marked distress and impair social and occupational functions.

A young woman comes in for a routine wellness examination. You notice that her vaginal walls have deep rugae and are slightly bluish in color. She also has a thicker white discharge. What should you suspect? A) Hypoxia B) Varicosities C) Pregnancy D) Sexually transmitted infection

C) Pregnancy You should suspect early pregnancy with a bluish discoloration of the vaginal walls and thicker discharge (Chadwick's sign). You may also notice a softening at the isthmus of the uterus on bimanual examination (Hegar's sign). New hypoxia in an otherwise asymptomatic patient would be unusual. While any unusual discharge should make one consider an STI, this is less likely.

Mrs. Glynn is 90 years old and lives alone. She is able to bathe, dress, prepare her food, and transfer from bed to chair independently. She has children in the area who help her withher medications and transportation needs. Which of the following is considered an instrumental activity of daily living? A) Bathing B) Dressing C) Preparing food D) Transferring from bed to chair

C) Preparing food Instrumental activities of daily living involve higher thought processes such as preparing food, whereas bathing, dressing, and transferring are considered physical activities of daily living.

A 26-year-old stewardess comes in for a third trimester prenatal visit. She has had prenatal care since her sixth week of pregnancy. She has no complaints today and her prenatal course has been unremarkable. Today her blood pressure and weight gain are appropriate and her urine is unremarkable. You have a first-year medical student shadowing you, so you ask the student to get Doptones and measure the patient's uterus in centimeters. The student promptly reports fetal heart tones of 140, but he is having difficulty obtaining the correct measurement. He knows one end of the tape goes over the uterine fundus. From what inferior anatomic position should the tape be placed? A) Vagina B) Clitoris C) Pubic symphysis D) Umbilicus

C) Pubic symphysis In most women, measuring from the symphysis to the uterine fundus is approximately the same and very predictable for dating purposes.

You arrive at the bedside of an elderly woman who has had a stroke, affecting her entire right side. She cannot speak (aphasia). You are supposed to examine her. You notice that the last examiner left her socks at the bottom of the bed, and although sensitive areas are covered by a sheet, the blanket is heaped by her feet at the bottom of the bed. What would you do next? A) Carry out your examination, focusing on the neurologic portion, and then cover her properly. B) Carry out your examination and let the nurse assigned to her "put her back together." C) Put her socks back on and cover her completely before beginning the evaluation. D) Apologize for the last examiner but let the next examiner dress and cover her.

C) Put her socks back on and cover her completely before beginning the evaluation. It is crucial to make an effort to make a patient comfortable. In this scenario, the patient can neither speak nor move well. Take a moment to imagine yourself in her situation. As a matter of respect as well as comfort, you should cover the patient appropriately and consider returning a little later to do your examination if you feel she is cold. While it is her nurse's job to keep her comfortable, it is also your responsibility, and you should do what you can. It is unacceptable to leave the patient in the same state in which you found her.

A 25-year-old type 1 diabetic clerk presents to the emergency room with shortness of breath and states that his blood sugar was 605 at home. You diagnose the patient with diabetic ketoacidosis. What is the expected pattern of breathing? A) Normal B) Rapid and shallow C) Rapid and deep D) Slow

C) Rapid and deep

Ms. Wright comes to your office, complaining of palpitations. While checking her pulse you notice an irregular rhythm. When you listen to her heart, every fourth beat sounds different. It sounds like a triplet rather than the usual "lub dup." How would you document your examination? A) Regular rate and rhythm B) Irregularly irregular rhythm C) Regularly irregular rhythm D) Bradycardia

C) Regularly irregular rhythm Because this unusual beat occurs every fourth set of heart sounds, it is regularly irregular. This is most consistent with ventricular premature contractions (or VPCs). This is generally a common and benign rhythm. An irregularly irregular rhythm is a classic finding in atrial fibrillation. The rhythm is very random in character. Bradycardia refers to the rate, not the rhythm.

A 6-month-old infant is brought in for a well check. It is noted his head circumference is off the chart and at a much higher percentile than was previously measured. What should you do next? A) Recommend a neurology consult. B) Order a CT of the head. C) Remeasure the circumference. D) Admit the child to the hospital for further workup.

C) Remeasure the circumference. It is difficult to obtain accurate measurements of a squirming infant. The first step would be to remeasure. Some recommend starting with three measurements and averaging or picking the middle measurement. Height is technically not measured until a child is standing, so infants' measurements are recorded as length.

Mrs. R. is a 92-year-old retired teacher who comes to your clinic accompanied by her daughter. You ask Mrs. R. why she came to your clinic today. She looks at her daughter and doesn't say anything in response to your question. This is an example of which type of challenging patient? A) Talkative patient B) Angry patient C) Silent patient D) Hearing-impaired patient

C) Silent patient This is one example of a silent patient. There are many possibilities for this patient's silence: depression, dementia, the manner in which you asked the question, and so on.

A mother brings in her 3-year-old son for a well-child check-up. She is concerned that he seems different in size from all of the other preschool boys. He was the product of an uneventful pregnancy and vaginal delivery. He has hit all of his developmental milestones on time. On examination he is 26 lbs (11.8 kg) and is 35 inches (89 cm) tall. Otherwise his examination is unremarkable. You give the correct education for his age and then discuss his size. For his age, what are his growth chart percentiles? A) Tall and heavy for his age (>95%) B) Average height and weight for his age (5 to 95%) C) Small and light for his age (<5%)

C) Small and light for his age (<5%) According to the growth charts, this child is less than the fifth percentile for both height and weight, indicating that he is small and light for his age. This can be from a growth hormone deficiency but is usually due to genetic factors (such as short, light parents). It is most important to follow the trend of growth. It is more significant if this child was previously at the 50th percentile for height and weight than if he has always been about the same percentile and following a line parallel to expected growth lines.

A 24-year-old mother who is a smoker and cocaine addict gave birth at 39 weeks to a 2,000-gram female infant who is in the neonatal intensive care unit. Using the Intrauterine Growth Curve chart, you determine whether the infant's weight is appropriate for her gestational age.In which category does the infant best fit? A) Large for gestational age B) Normal for gestational age C) Small for gestational age

C) Small for gestational age For a 39-week infant, any weight less than 2,500 grams would be considered small. Intrauterine growth retardation and low birth weight would be expected in a smoker who also abuses cocaine.

26-year-old violinist comes to your clinic, complaining of anxiety. He is a first chair violinist in the local symphony orchestra and has started having symptoms during performances, such as sweating, shaking, and hyperventilating. It has gotten so bad that he has thought about giving up his first chair status so he does not have to play the solo during one of the movements. He says that he never has these symptoms during rehearsals or when he is practicing. He denies having any of these symptoms at any other time. His past medical history is unremarkable. He denies any tobacco use, drug use, or alcohol abuse. His parents are both healthy. On examination you see a young man who appears worried. His vital signs and physical examination are unremarkable.What type of anxiety disorder best describes his situation? A) Panic disorder B) Specific phobia C) Social phobia D) Generalized anxiety disorder

C) Social phobia Social phobia is a marked, persistent fear of social or performance situations.

You are examining a 5-year-old before he begins school. You notice a systolic, gradeII/VI vibratory murmur over the LLSB and apex with normal S2 splitting. He has normal pulses as well. Which of the following is most likely? A) Tricuspid stenosis B) Mitral stenosis C) Still's murmur D) Venous hum

C) Still's murmur This description is consistent with Still's murmur, a very common and benign murmur of childhood. Tricuspid and mitral stenosis would be diastolic murmurs and the venous hum is usually not heard in this area. Further evaluation is usually not necessary.

A mother is upset because she was told by another provider that her child has a worrisome murmur. You listen near the clavicle and notice both a systolic and diastolic sound. You remember that diastolic murmurs are usually indicative of bad pathology. What would you do next? A) Cardiology referral B) Echocardiogram C) Supine examination D) Reassure the mother

C) Supine examination The next step would be to examine the patient in the supine position. If this is a venous hum, this murmur will resolve almost completely in the supine position. This is a very common phenomenon in school-aged children, particularly over the clavicle, but can also occur outside this range. Reassurance cannot be given without further examination, especially with a diastolic murmur. Cardiology referral and echocardiography are unnecessary if examination in the supine position reveals no murmur.

A woman in her 24th week of pregnancy notices she feels faint when lying down for a period. What would you suspect as a cause for this? A) Adrenal insufficiency B) Orthostatic hypotension C) Supine hypotensive syndrome D) Hypoglycemia

C) Supine hypotensive syndrome It is unusual to become lightheaded in the supine position. The gravid uterus can cause decreased blood return through the inferior vena cava. Orthostatic hypotension as seen in adrenal insufficiency and with moderate dehydration will cause these symptoms in the upright position. Hypoglycemia should not be positional.

Mr. Kelly comes to you today for a burning pain in his lower abdomen. This has gone on for 2 months. He has received radiation for prostatic cancer for the past quarter. What assumptions could you draw from this? A) This represents persistent pain. B) His pain reporting is likely to be unreliable. C) There are "red flags" present. D) He is depressed.

C) There are "red flags" present. This scenario is consistent with acute pain, although this may become persistent if it lasts beyond 3 months. The burning quality to the pain should be a red flag, along with descriptions of pain as "discomfort" or "soreness." Depressed affect or changes in posture or gait are also red flags. Studies have found that pain reporting in the elderly is accurate. Although depression may be present, we have no indications of it in this scenario

An 88-year-old retired piano teacher comes for evaluation of fatigue. You notice that her clothes are hanging loosely off her frame and that she has lost 15 pounds. She is unaware of this. Her husband of 63 years died a few months ago. You ask the patient to complete a Rapid Screen for Dietary Intake. Which of the following statements is considered to be part of this rapid screen? A) I eat more than two meals per day. B) I drink one glass of alcohol every day. C) Without wanting to, I have lost or gained 10 pounds in the last 6 months. D) I eat with at least one other person most of the time.

C) Without wanting to, I have lost or gained 10 pounds in the last 6 months. This is part of the Rapid Screen for Dietary Intake.

Despite having high BP readings in the office, Mr. Kelly tells you that his readings at home are much lower. He checks them twice a day at the same time of day and has kept a log. How do you respond? A) You diagnose "white coat hypertension." B) You assume he is quite nervous when he comes to your office. C) You question the accuracy of his measurements. D) You question the accuracy of your measurements.

C) You question the accuracy of his measurements. It is not uncommon to see differences in a patient's home measurements and your own in the office. Presuming that this is "white coat hypertension" can be dangerous because this condition is not usually treated. This allows for the effects of a missed diagnosis of hypertension to go unchecked. It is also very difficult to judge if a patient is outwardly nervous. You should always consider that your measurements are not accurate as well, but the fact that you and your staff are well-trained and perform this procedure on hundreds of patients a week makes this less likely. Ideally, you would ask the patient to bring in his BP equipment and take a simultaneous reading with you to make sure that he is getting an accurate reading.

Mr. Curtiss has a history of obesity, diabetes, osteoarthritis of the knees, HTN, and obstructive sleep apnea. His BMI is 43 and he has been discouraged by his difficulty in losing weight. He is also discouraged that his goal weight is 158 pounds away. What would you tell him? A) "When you get down to your goal weight, you will feel so much better." B) "Some people seem to be able to lose weight and others just can't, no matter how hard theytry." C) "We are coming up with new medicines and methods to treat your conditions every day." D) "Even a weight loss of 10% can make a noticeable improvement in the problems you mention."

D) "Even a weight loss of 10% can make a noticeable improvement in the problems you mention." Many patients trying to change a habit are overwhelmed by how far they are from their goal. As the proverb says: "A journey of a thousand miles begins with one step." Many patients find it empowering to know that they can achieve a small goal, such as a loss of 1 pound per week. They must be reminded that this process will take time and that slow weight loss is more successful long-term. Research has shown that significant benefits often come with even a 10% weight loss.

Claire's daughter brings her in today after Claire fell at her home. Which assessments are indicated at this time? A) Orthostatic vital signs B) Review of her medications C) Assessment of gait and balance D) All of the above

D) All of the above Falls are common in the elderly and can often result in serious injuries. When assessing the cause of falls, gait and balance should be checked first. Medication, particularly use of more than three, is associated with falls. Vision problems, lower-limb joint problems, and cardiovascular problems such as arrhythmias may be reasonable to search for. Orthostatic vital sign changes should be sought.

You are performing a young woman's first pelvic examination. You make sure to tell her verbally what is coming next and what to expect. Then you carry out each maneuver of the examination. You let her know at the outset that if she needs a break or wants to stop, this is possible. You ask several times during the examination, "How are you doing, Brittney?" What are you accomplishing with these techniques? A) Increasing the patient's sense of control B) Increasing the patient's trust in you as a caregiver C) Decreasing her sense of vulnerability D) All of the above

D) All of the above These techniques minimize the effects of transitions during an examination and empower the patient. Especially during a sensitive examination, it is important to give the patient as much control as possible.

An 85-year-old retired teacher comes to your office for evaluation of weakness. You obtain a complete history, perform a thorough physical examination, and order laboratory tests. You diagnose her with hyperthyroidism. Based on her age, which of the atypical symptoms of hyperthyroidism is more likely to be seen? A) Fatigue B) Weight loss C) Tachycardia D) Anorexia

D) Anorexia This is an atypical symptom of hyperthyroidism that is more likely to be seen in the older patient.

Adam is a very successful 15-year-old student and athlete. His mother brings him in today because he no longer studies, works out, or sees his friends. This has gone on for a month and a half. When you speak with him alone in the room, he states it "would be better if he were not here." What would you do next? A) Tell him that he has a very promising career in anything he chooses and soon he will feel better. B) Tell him that he needs an antidepressant and it will take about 4 weeks to work. C) Speak with his mother about getting him together more with his friends. D) Assess his suicide risk.

D) Assess his suicide risk. His lack of interest in usual activities and duration of symptoms should make you suspicious for depression. Despite his very successful academic and athletic performance, you should recognize this last phrase indicating suicide risk. You could ask if he has had thoughts about hurting himself and, if so, how he would carry this out. Ask about firearms and other weapons at home. Adam needs immediate psychiatric referral if these risks are found, or admission to the hospital for observation if referral is not available in a timely fashion.

You are beginning the examination of a patient. All of the following areas are important to observe as part of the General Survey except: A) Level of consciousness B) Signs of distress C) Dress, grooming, and personal hygiene D) Blood pressure

D) Blood pressure Blood pressure is a vital sign, not part of the General Survey

Which of the following changes are expected in vision as part of the normal aging process? A) Cataracts B) Glaucoma C) Macular degeneration D) Blurring of near vision

D) Blurring of near vision The lens loses its elasticity over time as part of the normal aging process, and the eye is less able to accommodate and focus on near objects; therefore, the patient will be expected to have blurring of near vision.

You are asked to perform a home safety assessment for an 87-year-old retired farmer who lives by himself. Which of the following is not considered to be an increased risk for falls? A) Loose electrical cords B) Slippery or irregular surfaces C) Chairs at awkward angles D) Bright lighting

D) Bright lighting Bright lighting is a recommendation to improve an older person's ability to see all possible things that could result in a fall.

You are running late after your quarterly quality improvement meeting at the hospital and have just gotten paged from the nurses' station because a family member of one of your patients wants to talk with you about that patient's care. You have clinic this afternoon and are double-booked for the first appointment time; three other patients also have arrived and are sitting in the waiting room. Which of the following demeanors is a behavior consistent with skilled interviewing when you walk into the examination room to speak with your first clinic patient? A) Irritability B) Impatience C) Boredom D) Calm

D) Calm The appearance of calmness and patience, even when time is limited, is the hallmark of a skilled interviewer.

You are examining an infant in the nursery and notice a soft bump over the posterior right side of the skull. It is not evident on the left. What does this represent? A) Caput succedaneum B) Plagiocephaly C) Craniosynostosis D) Cephalohematoma

D) Cephalohematoma Cephalohematoma represents bleeding under the periosteum, which is why this lesion does not cross the midline. The blood can contribute to neonatal jaundice as it breaks down. Caput succedaneum is commonly seen as a spongy mass over the vertex, particularly when vacuum extraction is used. Craniosynostosis describes a premature closure of bony skull sutures, and plagiocephaly is a flattening of the parieto-occipital region on one side of the skull, which is frequently thought to be positional.

A patient complains of knee pain on your arrival in the room. What should be your first sentence be after greeting the patient? A) How much pain are you having? B) Have you injured this knee in the past? C) When did this first occur? D) Could you please describe what happened?

D) Could you please describe what happened? When looking into a complaint, it is best to start with an invitation for the patient to tell you in his or her own words. More specific questions should be used later in the interview to fill in any gaps.

An 89-year-old retired school principal comes for an annual check-up. She would like to know whether or not she should undergo a screening colonoscopy. She has never done this before. Which of the following factors should not be considered when discussing whether she should go for this screening test? A) Life expectancy B) Time interval until benefit from screening accrues C) Patient preference D) Current age of patient

D) Current age of patient The current age of the patient is not as important as her actual life expectancy and current health status.

Which of the following accompanies decreased ovarian function? A) Increased sleep B) Diminution of sexual interest C) Enlargement of the clitoris D) Decrease in vaginal secretions

D) Decrease in vaginal secretions Menopause, or the cessation of menses for 1 year, commonly occurs in the late 40s to early 50s. Many experience hot flashes, sweating, chills, anxiety, decreased sleep, and urge incontinence. Dyspareunia is common secondary to decreased vaginal secretions. Sexual interest does not normally decrease. The clitoris and length of the vaginal vault decrease in response to decreased estrogen.

A 70-year-old retired auto mechanic comes to your office because his neighbor is concerned about his memory. The patient himself admits to misplacing his keys more often and forgets what he is supposed to buy from the grocery store and where he has parked the car. He denies getting lost in familiar places. Upon further questioning, he states that his wife of 40 years died 8 months ago; his three children live in three different states; and he has limited his activities because the people he interacted with were "his wife's friends, not his." He drinks a six-pack of beer daily; he does not smoke or use illicit drugs. You perform a mini-mental state examination and obtain a total score of 24 out of 28. Based on this information, what is your most likely diagnosis? A) Benign forgetfulness B) Dementia C) Meningitis D) Depression

D) Depression The patient has symptoms of depression: His wife died, he has no real social support system, and he has isolated himself from his usual activities. He also drinks a considerable amount of alcohol on a daily basis, which can further depress his mood. Depression can masquerade as dementia in the elderly and must be considered in a patient with memory loss.

A pregnant woman is concerned by the recent onset of a midline swelling. It is soft and nontender. What does this represent? A) Linea nigra B) Chadwick's sign C) Round ligament pain D) Diastasis recti

D) Diastasis recti In advanced pregnancy, muscle tone diminishes, which may aid in the separation of the rectus abdominis muscles. This is a benign finding and does not usually cause other symptoms. You may palpate the fetus well through this opening. Linea nigra is a hyperpigmentation along the midline. Chadwick's sign is the bluish tinge to the cervix and vaginal walls seen early in pregnancy, and round ligament pain occurs as the uterus enlarges. This discomfort is usually found in the right more often than the left.

Which of the following is worrisome in Melissa, a woman in her 26th week of pregnancy? A) Generalized hair loss B) A hyperpigmented rash over the maxillary region bilaterally C) Nosebleeds D) Facial edema

D) Facial edema Generalized hair loss, hyperpigmented maxillary rash (chloasma), and nosebleeds are usually benign and common in pregnancy. Facial edema after the 24th week of gestation may indicate gestational hypertension.

Lucille is in her 24th week. You notice a new onset of high blood pressure readings. Today's value is 168/96. Her urine is normal. What do you suspect? A) Preeclampsia B) Chronic hypertension C) Supine hypotensive syndrome D) Gestational hypertension

D) Gestational hypertension Because Lucille's BP is greater than 140/90 with onset after the 20th week and no proteinuria, this is gestational hypertension. Chronic hypertension, while the same BP cutoff is used, is present before the 20th week. If this patient had accompanying proteinuria, preeclampsia could be considered. Supine hypotensive syndrome does not cause hypertension.

On routine screening you notice that the cup-to-disc ratio of the patient's right eye is 1:2. What ocular condition should you suspect? A) Macular degeneration B) Diabetic retinopathy C) Hypertensive retinopathy D) Glaucoma

D) Glaucoma This cup-to-disc ratio means that the cup takes up 50% of the disc, which is abnormally large. This is usually an indication of glaucoma, which is a common cause of visual loss in the elderly. The cup-to-disc changes are not seen in diabetes, hypertension, or macular degeneration. Many elderly do not have regular eye examinations and are not screened for glaucoma.

A mother brings her 16-month-old son in for an evaluation. She is afraid he is not meeting his developmental milestones and wants to know if he should be sent to therapy. He was the product of an uneventful pregnancy and a spontaneous vaginal delivery. His Apgar scores were 7 and 9. Until reaching a year old the mother believes he was hitting his milestones appropriately. You decide to administer the Denver Developmental Screening Test. You find that he is using a spoon to eat with and can take off his own shoes and shirt. He can build a tower of two cubes and dump raisins. His vocabulary consists of at least 10 words. He can stand alone and stoop and recover, but he is unable to walk without holding onto someone's hand.What type of developmental delay does he have? A) Personal/social B) Fine motor C) Language D) Gross motor

D) Gross motor By 16 months a child should be able to walk unaided and even walk backwards and run. This child was referred to physical therapy and did well.

A 67-year-old retired janitor comes to the clinic with his wife. She brought him in because she is concerned about his weight loss. He has a history of smoking 3 packs of cigarettes a day for 30 years, for a total of 90 pack-years. He has noticed a daily cough for the past several years, which he states is productive of sputum. He came into the clinic approximately 1 year ago, and at that time his weight was 140 pounds. Today, his weight is 110 pounds.Which one of the following questions would be the most important to ask if you suspect that he has lung cancer? A) Have you tried to force yourself to vomit after eating a meal? B) Do you have heartburn/indigestion and diarrhea? C) Do you have enough food to eat? D) Have you tried to lose weight?

D) Have you tried to lose weight? This is important: If the patient hasn't tried to lose weight, then this weight loss is inadvertent and poses concern for a neoplastic process, especially given his smoking history.

A 50-year-old body builder is upset by a letter of denial from his life insurance company. He is very lean but has gained 2 pounds over the past 6 months. You personally performed his health assessment and found no problems whatsoever. He says he is classified as "high risk" because of obesity. What should you do next? A) Explain that even small amounts of weight gain can classify you as obese. B) Place him on a high-protein, low-fat diet. C) Advise him to increase his aerobic exercise for calorie burning. D) Measure his waist.

D) Measure his waist. The patient most likely had a high BMI because of increased muscle mass. In this situation, it is important to measure his waist. It is most likely under 40 inches, which makes obesity unlikely (even to an insurance company). It is important that you personally contact the company and explain your reasoning. Be prepared to back your argument with data. A special diet is unlikely to be of much use, and more aerobic exercise, while probably a good idea for most, is redundant for this individual.

27. A 15-month-old is brought to you for a fever of 38.6 degrees Celsius and fussiness. The ear examination is as follows: external ear, normal appearance and no tenderness with manipulation; canal, normal diameter without evidence of inflammation; tympanic membrane, bulging, erythematous, and opaque. Insufflation is deferred due to pain. What is your diagnosis? A) Otitis externa B) Cholesteatoma C) Ruptured tympanic membrane D) Otitis media

D) Otitis media There is no inflammation of the outer ear, including the canal, thus excluding otitis externa. Cholesteatoma is a painless white lesion behind the TM. There is no drainage from the TM; thus, rupture is unlikely. This is a classic description of otitis media. Many examiners will forego insufflation if the diagnosis is clear, because this can cause discomfort in an already uncomfortable ear.

In obtaining a history, you note that a patient uses the word "largely" repeatedly, to thepoint of being a distraction to your task. A) Clanging B) Echolalia C) Confabulation D) Perseveration

D) Perseveration Perseveration is the repetition of words or ideas. Echolalia differs in that the patient repeats what is said to him. Clanging is the repetition of the same sounds in different words. Confabulation is making up a story in response to a question. This is sometimes seen in chronic alcohol use with Korsakoff's syndrome.

Which of the following is commonly seen in aging men? A) Erectile dysfunction in 20% of all men B) Testicles ride higher within the scrotum C) Strong response to visual erotic cues D) Persistent sexual interest

D) Persistent sexual interest Erectile dysfunction affects about half of elderly men but sexual interest generally remains intact. A decrease in sexual interest may indicate other problems such as depression.Visual cues become less important and tactile stimulation more important. The testicles are positioned lower in the scrotum.

A patient is describing a very personal part of her history very quickly and in great detail. How should you react to this? A) Write down as much as you can, as quickly as possible. B) Ask her to repeat key phrases or to pause at regular intervals, so you can get almost everyword. C) Tell her that she can go over the notes later to make sure they are accurate. D) Push away from the keyboard or put down your pen and listen.

D) Push away from the keyboard or put down your pen and listen. This is a common event in clinical practice. It is much more important to listen actively with good eye contact at this time than to document the story verbatim. You want to minimize interruption (e.g., answer B). It is usually not appropriate to ask a patient to go over the written notes, but it would be a good idea to repeat the main ideas back to her. You should be certain she has completed her story before doing this. By putting down your pen or pushing away from the keyboard, you let the patient know that her story is the most important thing to you at this moment.

A 19-year-old college student, Todd, is brought to your clinic by his mother. She is concerned that there is something seriously wrong with him. She states for the past 6 months his behavior has become peculiar and he has flunked out of college. Todd denies any recent illness or injuries. His past medical history is remarkable only for a broken foot. His parents are both healthy. He has a paternal uncle who had similar symptoms in college. The patient admits to smoking cigarettes and drinking alcohol. He also admits to marijuana use but none in the last week. He denies using any other substances. He denies any feelings of depression or anxiety. While speaking with Todd and his mother you do a complete physical examination, which is essentially normal. When you question him on how he is feeling, he says that he is very worried that Microsoft has stolen his software for creating a better browser. He tells you he has seen a black van in his neighborhood at night and he is sure that it is full of computer tech workers stealing his work through special gamma waves. You ask him why he believes they are trying to steal his programs. He replies that the technicians have been telepathing their intents directly into his head. He says he hears these conversations at night so he knows this is happening. Todd's mother then tells you, "See, I told you . . . he's crazy. What do I do about it?"While arranging for a psychiatry consult, what psychotic disorder do you think Todd has? A) Schizoaffective disorder B) Psychotic disorder due to a medical illness C) Substance-induced psychotic disorder D) Schizophrenia

D) Schizophrenia Schizophrenia generally occurs in the late teens to early 20s. It often is seen in other family members, as in this case. Symptoms must be present for at least 6 months and must have at least two features of (1) delusions (e.g., Microsoft is after his programs), (2) hallucinations (e.g., technicians sending telepathic signals), (3) disorganized speech, (4) disorganized behavior, and (5) negative symptoms such as a flat affect.

Jenny is one of your favorite patients who usually shares a joke with you and is nattily dressed. Today she is dressed in old jeans, lacks makeup, and avoids eye contact. To what do you attribute these changes? A) She is lacking sleep. B) She is fatigued from work. C) She is running into financial difficulty. D) She is depressed.

D) She is depressed. It is important to use all of your skills and memory of an individual patient to guide your thought process. She is not described as sleepy. Work fatigue would most likely not cause avoidance of eye contact. Financial difficulties would not necessarily deplete a nice wardrobe. It is most likely that she is depressed or in another type of difficulty.

Susanne is a 27 year old who has had headaches, muscle aches, and fatigue for the last 2 months. You have completed a thorough history, examination, and laboratory workup but have not found a cause. What would your next action be? A) A referral to a neurologist B) A referral to a rheumatologist C) To tell the patient you can't find anything D) To screen for depression

D) To screen for depression Although you may consider referrals to help with the diagnosis and treatment for this patient, screening is a time-efficient way to recognize depression. This will allow her to be treated more expediently. You may tell the patient you have not found an answer yet, but you must also tell her that you will not stop looking until you have helped her.

A woman has a positive pregnancy test and comes to you with left lower quadrant pain. On bimanual examination, you feel a tender mass. Which of the following should you suspect? A) Threatened abortion B) Appendicitis C) Ovarian cyst D) Tubal pregnancy

D) Tubal pregnancy Lower quadrant pain in a young woman could represent any of these possibilities. A positive HCG and left-sided but not right-sided pain make appendicitis less likely. The presence of an extrauterine mass makes threatened abortion less likely.

A patient tells you about her experience with prolonged therapy for her breast cancer. You comment, "That must have been a very trying time for you." What is this an example of? A) Reassurance B) Empathy C) Summarization D) Validation

D) Validation This is an example of validation to legitimize her emotional experience. "Now that you have had your treatment, you should not have any further troubles" is an example of reassurance. "I understand what you went through because I am a cancer survivor myself" is an example of empathy. "So, you have had a lumpectomy and multiple radiation treatments" is an example of summarization as applied to this vignette.

The following information is best placed in which category? "The patient had a stent placed in the left anterior descending artery (LAD) in 1999." a. adult illnesses. b. surgeries. c. obstetrics/gynecology. d. psychiatric.

a. adult illnesses. The adult illness category is reserved for chronic illnesses, significant hospitalization, significant injuries, and significant procedures. A stent is a major procedure but does not involve a surgeon.

The following information is best placed in which category? "The patient was treated for an asthma exacerbation in the hospital last year; the patient has never been intubated." a. adult illnesses. b. surgeries. c. obstetrics/gynecology. d. psychiatric.

a. adult illnesses. This is information about a significant hospitalization and should be placed in the adult illnesses section. If the patient is being seen for an asthma exacerbation, you may consider placing this information in the present illness section, because it related to the chief complain at that visit.

The following information is recorded in the health history: "I feel really tired." Which category does it belong to? a. chief complaint. b. present illness. c. personal and social history. d. review of systems.

a. chief complaint. The chief complain is an attempt to quote the patient's own words, as long as they are suitable to print. It is a brief, like a headline, and further details should be sought in the present illness section.

You have recently returned from a medical missions trip to sub-Saharan Africa, where you learned a great deal about malaria. You decide to use some of the same questions and maneuvers in your "routine" when examining patients in the midwestern United States. You are disappointed to find that despite getting some positive answers and findings, on further workup, none of your patients has malaria except one, who recently emigrated from Ghana. How should you next approach these questions and maneuvers? a. continue asking these questions in a more selective way. b. stop asking these questions, because they are low yield. c. question the validity of the questions. d. ask these questions of all your patients.

a. continue asking these questions in a more selective way. The predictive value of a positive finding depends upon the prevalence of a given disease in a population. The prevalence of malaria in the Midwest is almost zero, except in people immigrating from areas of high prevalence. You will waste time and resources applying these questions and maneuvers to all patients. It would be wise to continue applying what you learned to those who are from areas of high prevalence of a given disease. Likewise, physicians from Ghana should not ask about signs or symptoms of multiple sclerosis, as it is found almost exclusively in northern latitudes. You will learn to tailor your examination to the population you are serving.

A patient presents for evaluation of a sharp, aching chest pain which increases with breathing. Which anatomic area would you localize the symptom to? a. musculoskeletal. b. reproductive. c. urinary. d. endocrine.

a. musculoskeletal. Chest pain may be due to a musculoskeletal condition, such as costochondritis or intercostal muscle cramp. This would be worsened by motion of the chest wall. Pleuritic chest pain is also a sharp chest pain which increases with a deep breath. This type of pain can occur with inflammation of the pleura from pneumonia or other conditions and pulmonary embolus.

A 62-year-old teach presents to the clinic for evaluation of the following symptoms: fever, headache, sinus congestion, sore throat, green nasal discharge, and cough. This cluster of symptoms is best explained by: a. one disease process. b. more than one disease process.

a. one disease process. This cluster of symptoms is most consistent with sinusitis. The chance that all of these symptoms are cause by multiple synchronous conditions in the same patient is much less than the possibility of having on problem which accounts for all of them.

Ms. Washington is a 67-year-old who had a heart attack last month. Now she complains of shortness of breath and not being able to sleep in a flat position (orthopnea). On examination you note increased jugular venous pressure, an S3 gallop, crackles low in the lung fields, and swollen ankles (edema). This is an example of a: a. pathophysiologic problem b. psychopathologic problem

a. pathophysiologic problem This is an example of a pathophysiologic problem because Ms. Washington's symptoms are consistent with a pathophysiologic process. The heart attack reduced the ability of her heart to handle her volume status and subsequently produced the many features of CHF.

Is the following information subjective or objective? Mr. M has shortness of breath that has persisted for the past 10 days; it is worse with activity and relieved by rest. a. subjective. b. objective.

a. subjective. This is information given by the patient about the circumstances of his chief complain.

You are growing fatigued of performing a maneuver on examination because you have never found a positive and are usually pressed for time. How should you next approach this maneuver? a. use this test when you have a higher suspicion for a certain correlating condition. b. omit this test from future examinations. c. continue doing the test, but rely more heavily on laboratory work and diagnostics. d. continue performing it on all future examinations.

a. use this test when you have a higher suspicion for a certain correlating condition. his is an example of a specific test that lacks sensitivity. With this scenario, when you finally find a positive, you might be very certain that a given condition is present.We generally develop our examinations to fit our clinical experiences. Sensitive tests are performed routinely on the screening examination, while specific tests are usually saved for the detailed or "branched" examinations. Branched examinations are further maneuvers we can perform to investigate positive findings on our screening examinations. Save this type of maneuver to confirm your hypothesis. All of this information also applies to history questions.

A 22-year-old advertising copywriter presents for evaluation of joint pain. The pain is new, located in the wrists and fingers bilaterally, with some subjective fever. The patient denies a rash; she also denies recent travel or camping activities. She has a family history significant for rheumatoid arthritis. Based on this information, which of the following pathologic processes would be the most correct? a. infectious. b. inflammatory. c. hematologic. d. traumatic.

b. inflammatory. The description is most consistent with an inflammatory process, although all the other etiologies should be considered. Lyme disease which commonly causes arthritis, hemophilia is a hematologic condition which can cause bleeding in the joins, and trauma can obviously cause joint pain. Your clinical reasoning skills are important for sorting through all of the data to arrive at the most likely conclusion.

A 55-year-old data entry operator comes to the clinic to establish care. She has the following symptoms: headache, neck pain, sinus congestion, sore throat, ringing in ears, sharp brief chest pains at rest, burning abdominal pain with spicy foods, constipation, urinary frequency that is worse with coughing and sneezing, and swelling in legs. This cluster of symptoms is explained by: a. one disease process. b. more than one disease process.

b. more than one disease process. The patient appears to have several possible conditions: allergic rhinitis, arthritis, conductive hearing loss, pleuritic chest pains, heartburn, stress urinary incontinence, and venous stasis, among other conditions. Although we always try, it is very difficult to assign all of these symptoms to one cohesive diagnosis.

Is the following information subjective or objective? Mr. M has a respiratory rate of 32 and a pulse rate of 120. a. subjective. b. objective.

b. objective. This is a measurement obtained by the examiner, so it is considered objective data.

The following information is recorded in the health history: "The patient has had abdominal pain for 1 week. The pain lasts for 30 minutes at a time; it comes and goes. The severity is 7 to 9 on a scale of 1 to 10. It is accompanied by nausea and vomiting. It is located in the mid-epigastric area." Which of these categories does it belong to? a. chief complain. b. present illness. c. person and social history. d. review of systems.

b. present illness. This information describes the problem of abdominal pain, which is the present illness. The interviewer has obtained the location, timing, severity, and associated manifestations of the pain. The interviewer will still need to obtain info concerning the quality of the pain, the setting in which it occurred, and the factors that aggravate and alleviate the pain. You will notice that it does not include portions of the pertinent review of systems, but because it related directly to the complain, it is included in the history of present illness.

On the way to see your next patient, you glance at the calendar and make a mental note to buy a Mother's Day card. Your patient is Ms. Hernandez, a 76-year-old widow who lost her husband in May, two years ago. She comes in today with a headache, abdominal pain, and general malaise. This happened once before, about a year ago, according to your detailed office notes. You have done a thorough evaluation but are unable to arrive at a consistent picture to tie these symptoms together. This is an example of a: a. pathophysiologic problem. b. psychopathologic problem.

b. psychopathologic problem. It is not uncommon to experience psychopathologic symptoms around the anniversary of a traumatic event. The time of year and the lack of an obvious connection between Ms. Hernandez's symptoms would make you consider this as a possibility. You will note that although this might have been an early consideration in your hypothesis generation, it is key to convince yourself that there is not a physiologic explanation for these symptoms, by performing a careful history and examination.

The following information is best placed in which category? "The patient has had three cesarean sections." a. adult illnesses. b. surgeries. c. obstetrics/gynecology. d. psychiatric.

b. surgeries. A cesarian section is a surgical procedure. Approximate dates or the ages of the patient at the time of the surgery should also be recorded.

The components of the health history include all of the following except which one? a. review of systems. b. thorax and lungs. c. present illness. d. personal and social items.

b. thorax and lungs. The thorax and lungs are part of the physical examination, not part of the health history.

For which of the following patients would a comprehensive health history be appropriate? a. a new patient with the chief complaint of "I sprained my ankle." b. an established patient with the chief complain of "I have an upper respiratory infection." c. a new patient with the chief complaint of "I am here to establish care." d. a new patient with the chief complaint of "I cut my hand."

c. a new patient with the chief complaint of "I am here to establish care." This patient is here to establish care, and because she is new to you, a comprehensive health history is appropriate.

A 37-year-old nurse comes for evaluation of colicky right upper quadrant abdominal pain. The pain is associated with nausea and vomiting and occurs 1 to 2 hours after eating greasy foods. Which one of the following physical examination description would be most consistent with the diagnosis of cholecystitis? a. abdomen is soft, nontender, and non distended, without hepatosplenomegaly or masses. b. abdomen is soft and tender to palpation in the right lower quadrant, without rebound or guarding. c. abdomen is soft and tender in the right upper quadrant with inspiration, to the point of stopping inspiration, and there is no rebound or guarding. d. abdomen is soft and tender to palpation in the mid-epigastric area, without rebound or guarding.

c. abdomen is soft and tender in the right upper quadrant with inspiration, to the point of stopping inspiration, and there is no rebound or guarding. In cholecystitis, the pain, which originates from the gallbladder, is located in the right upper quadrant. Severity of pain with inspiration that is sufficient to stop further inhalation is also known as Murphy's sign, which if present, is further indicative of inflammation of the gallbladder.

A 15-year-old high school sophomore comes to the clinic for evaluation of a 3-week history of sneezing; itchy, watery eyes; clear nasal discharge; ear pain; and nonproductive cough. Which is the most likely pathologic process? a. infection. b. inflammation. c. allergic. d. vascular.

c. allergic. This description is most consistent with allergic rhinitis.

A patient comes to the emergency room for evaluation of shortness of breath. To which anatomic region would you assign the symptom? a. reproductive. b. urinary. c. cardiac. d. hematologic.

c. cardiac. Cardiac disorders such as congestive heart failure are the most likely on this list to result in shortness of breath. There are cases within the other categories which may also result in shortness of breath, such as anemia in the hematologic category, pregnancy in the reproductive category, or sepsis with UTI in the urinary category. This demonstrates the "tension" in clinical reasoning between making sure all possibilities are covered, while still being able to pick the most likely cause.

A patient presents for evaluation of a cough. Which of the following anatomic regions can be responsible for a cough? a. ophthalmologic. b. auditory. c. cardiac. d. endocrine.

c. cardiac. The cardiac system can cause a cough if the patient has congestive heart failure. This results in fluid buildup in the lungs, which in turn can cause a cough that produces pink, frothy sputum. A foreign body in the ear may also cause a cough by stimulating Arnold's branch of the vagus nerve, but this is less likely to be seen clinically than heart failure.

You are excited about a positive test finding you have just noticed on physical examination of your patient. You go on to do more examination, laboratory work, and diagnostic tests, only to find that there is no sign of the disease you thought would correlate with the finding. This same experience happens several times. What should you conclude? a. consider not doing this test routinely. b. use this test when you have a higher suspicion for a certain correlating condition. c. continue using the test, perhaps doing less laboratory work and diagnostics. d. omit this test from future examinations.

c. continue using the test, perhaps doing less laboratory work and diagnostics. This is an example of a sensitive physical finding that lacks specificity. This does not make this a useless test, because the purpose of a screening physical is to find disease. This finding made you consider the associated condition as one of your hypotheses, and this in itself has value. Other possibilities are that you may be doing the maneuver incorrectly or using it on the wrong population. It is important to ask for hands-on help from your instructor when you have a question about a maneuver. Make sure that your information about the maneuver comes from a reliable source as well. All of this information also applies to history questions.

A 47-year-old contractor presents for evaluation of neck pain, which has been intermittent for several years. He normally takes OTC medication to ease the pain, but this time they haven't worked as well and he still has discomfort. He recently wallpapered the entire second floor in his house, which caused him great discomfort. The pain resolved with rest. He denies fever, chills, rash, upper respiratory symptoms, trauma, or injury to the neck. Based on this description, what is the most likely pathologic process? a. infectious. b. neoplastic. c. degenerative. d. traumatic.

c. degenerative. The description is most consistent with degenerative arthritis in the neck. The patient has had intermittent symptoms and the questions asked to elicit pertinent negative and positive findings are negative for infectious, traumatic, or neoplastic disease.

Mr. Larson is a 42-year-old widowed father of two children, ages 4 and 11. He works in a sales office to support his family. Recently he has injured his back and you are thinking he would benefit from physical therapy, three times a week, for an hour per session. What would be your next step? a. write the physical therapy prescription. b. have your office staff explain directions to the physical therapy center. c. discuss the plan with Mr. Larson. d. tell Mr. Larson that he will be going to physical therapy three times a week.

c. discuss the plan with Mr. Larson. You should discuss your proposed plan with the patient before implementing it. In this case, you and Mr. Larson will need to weigh the benefit of physical therapy against the ability to provide for his family. You may need to consider other ways of helping the patient, perhaps through prescribed back exercises he can do at home. It is a common mistake to implement a plan without coming to an agreement with the patient first.

A 19-year-old college student presents to the emergency room with fever, headache, and neck pain/stiffness. She is concerned about the possibility of meningococcal meningitis. Several of her dorm mates have been vaccinated, but she hasn't been. Which of the following physical examination descriptions is most consistent with meningitis? a. head is normocephalic and atraumatic, fundi with sharp discs, neck supple with full range of motion. b. head is normocephalic and atraumatic, fundi with sharp discs, neck with paraspinous muscle spasms and limited range of motion to the right. c. head is normocephalic and atraumatic, fundi with blurred disc margins, neck tender to palpation, unable to perform range of motion. d. head is normocephalic and atraumatic, fundi with blurred disc margins, neck supple with full range of motion.

c. head is normocephalic and atraumatic, fundi with blurred disc margins, neck tender to palpation, unable to perform range of motion. Blurred disc margins are consistent with papilledema, and neck tenderness and lack of range of motion are consistent with neck stiffness, which in this scenario is likely to be caused by meningeal inflammation. Later, you will learn about Kernig's and Brudzinski's signs, which are helpful in testing for meningeal irritation on examination.

The following information is recorded in the health history: "The patient completed 8th grade. He currently lives with his wife and two children. He works on old cars on the weekend. He works in a glass factory during the week." Which category does it belong to? a. chief complain. b. present illness. c. personal and social history. d. review of systems.

c. personal and social history. Personal and social history info includes educational level, family of origin, current household status, personal interests, employment, religious beliefs, military history, and lifestyle (including diet and exercise habits; use of alcohol, tobacco, and/or dugs; and sexual preferences and history).

You are seeing an elderly man with multiple complaints. He has chronic arthritis, pain from an old war injury, and headaches. Today he complains of these pains, as well as dull chest pain under his sternum. What would the order of priority be for your problem list? a. arthritis, war injury pain, headaches, chest pain b. war injury pain, arthritis, headaches, chest pain c. headaches, arthritis, war injury pain, chest pain d. chest pain, headaches, arthritis, war injury pain

d. chest pain, headaches, arthritis, war injury pain The problem list should have the most active and serious problem first. This new complaint of chest pain is almost certainly a higher priority than his other, more chronic problems.

The following information is recorded in the health history: "Patient denies chest pain, palpitations, orthopnea, and paroxysmal nocturnal dyspnea." Which category does it belong to? a. chief complaint. b. present illness. c. personal and social history. d. review of systems.

d. review of systems. Review of systems documents the present or absence of common symptoms related to each major body system. The absence of cardiac symptoms is listed in the above example.

Steve has just seen a 5-year-old girl who sneezes when exposed to cats. The patient's family history is positive for asthma. You think the child most likely has asthma. What have you just accomplished? a. you have tested your hypothesis. b. you have developed a plan. c. you have established a working diagnosis. d. you have created a hypothesis.

d. you have created a hypothesis. As you go through a history and exam, you will start to generate ideas to explain the patient's symptoms. It is best to keep an open mind and make as many hypotheses as you can to avoid missing a possibility. A common mistake is to latch onto one idea too early. Once you have committed your mind to a diagnosis, it is difficult to change to another. To think about looking for wheezes on examination would be an example of testing your new hypothesis. Starting a patient on an inhaled medicine would be a plan. It is too early to commit to a working diagnosis, given the amount of info you have gathered.


संबंधित स्टडी सेट्स

EXSC 301 Final Exam/ heart disease & obesity (pt. 1)

View Set

FHA Loans and the Consumer Workshops

View Set

Chapter 36: Managements with Patients Immune Deficiency Disorders

View Set

Chapter 11: variety, emphasis, harmony, and unity

View Set